70
1 Prelim IAS Test Series (2019) GS Test 5 (23.12.2018) History 1 and Current Affairs (July 2018) Answer Key & Exp for GS Prelim Test-5 held on 23 rd Dec’ 2018 1. Consider the following statements about the Mauryan economy: (1) Slavery was absent during the Mauryan reign but emerged during the time of the Kushanas. (2) The use of currency as a medium of exchange was popular during the Mauryan times. (3) There was remarkable growth of settled agriculture during the Mauryan times. Which of the statements given above is/are correct? (a) 2 only (b) 1 and 3 only (c) 2 and 3 only (d) 1, 2 and 3 Answer: (c) Explanation: Statement 1 is incorrect. It is known that the Mauryans owned large farms known as ‘Sita’ which were worked by numerous slaves and hired labourers. Also, Kautilya in his Arthashastra mentioned elaborately about slaves, both men and women. Statement 2 is correct. The use of currency which began in the pre-Mauryan period became a fairly common feature of the Mauryan period. Currency was used to trade goods and the government paid its officers in cash. It is evident from a large number of coins of Mauryan age found at many places like U.P. and Bihar. Statement 3 is correct. Large-scale clearing of land by the State as well as the cultivation of Crown land under the direct supervision of State’s officers led to an unprecedented growth of settled agriculture, especially in the Gangetic valley. Source: Vajiram and Ravi Yellow book- “Ancient India”- Chapter 6, Page 48-49. 2. Consider the following terms used during the Mauryan times: Term Used for 1. Amatyas : Civil servants 2. Sannidhata : Chief Treasury Officer 3. Gudhapurushas : Secret Agents Which of the pairs given above is/are correctly matched? (a) 1 only (b) 1 and 3 only (c) 2 and 3 only (d) 1, 2 and 3 https://telegram.me/UPSCMaterials https://telegram.me/FreeUPSCMaterials https://telegram.me/MaterialforExam

Answer Key & Exp for GS Prelim Test-5 held on 23rd …...1 Prelim IAS Test Series (2019) – GS Test 5 (23.12.2018) History 1 and Current Affairs (July 2018) Answer Key & Exp for GS

  • Upload
    others

  • View
    14

  • Download
    0

Embed Size (px)

Citation preview

Page 1: Answer Key & Exp for GS Prelim Test-5 held on 23rd …...1 Prelim IAS Test Series (2019) – GS Test 5 (23.12.2018) History 1 and Current Affairs (July 2018) Answer Key & Exp for GS

1

Prelim IAS Test Series (2019) – GS Test 5 (23.12.2018) History 1 and Current Affairs (July 2018)

Answer Key & Exp for GS Prelim Test-5 held on 23rd Dec’ 2018

1. Consider the following statements about the Mauryan economy: (1) Slavery was absent during the Mauryan reign but emerged during the time of

the Kushanas. (2) The use of currency as a medium of exchange was popular during the

Mauryan times. (3) There was remarkable growth of settled agriculture during the Mauryan times. Which of the statements given above is/are correct? (a) 2 only (b) 1 and 3 only (c) 2 and 3 only (d) 1, 2 and 3

Answer: (c) Explanation: Statement 1 is incorrect. It is known that the Mauryans owned large farms known as ‘Sita’ which were worked by numerous slaves and hired labourers. Also, Kautilya in his Arthashastra mentioned elaborately about slaves, both men and women. Statement 2 is correct. The use of currency which began in the pre-Mauryan period became a fairly common feature of the Mauryan period. Currency was used to trade goods and the government paid its officers in cash. It is evident from a large number of coins of Mauryan age found at many places like U.P. and Bihar. Statement 3 is correct. Large-scale clearing of land by the State as well as the cultivation of Crown land under the direct supervision of State’s officers led to an unprecedented growth of settled agriculture, especially in the Gangetic valley. Source: Vajiram and Ravi Yellow book- “Ancient India”- Chapter 6, Page 48-49. 2. Consider the following terms used during the Mauryan times:

Term Used for

1. Amatyas : Civil servants

2. Sannidhata : Chief Treasury Officer

3. Gudhapurushas : Secret Agents

Which of the pairs given above is/are correctly matched? (a) 1 only (b) 1 and 3 only (c) 2 and 3 only (d) 1, 2 and 3

https://telegram.me/UPSCMaterials https://telegram.me/FreeUPSCMaterials https://telegram.me/MaterialforExam

Page 2: Answer Key & Exp for GS Prelim Test-5 held on 23rd …...1 Prelim IAS Test Series (2019) – GS Test 5 (23.12.2018) History 1 and Current Affairs (July 2018) Answer Key & Exp for GS

2

Prelim IAS Test Series (2019) – GS Test 5 (23.12.2018) History 1 and Current Affairs (July 2018)

Answer: (d) Explanation: Pair 1 is correctly matched. Amatyas were civil servants who filled the highest administrative and judicial appointments. Their qualifications and methods of selection were laid out by Kautilya. Their role and functions were very important as all the government work proceeded from them. Pair 2 is correctly matched. The revenue department was manned by Samharta (in-charge of collection of all revenue) and Sannidhata (Chief Treasury Officer). Pair 3 is correctly matched. The espionage department consisted of Gudhapurushas (secret agents) under the control of Mahamatyapasarpa. Additional Information:

Term: Used for:

Mantrins Ministers

Mahamatyas Civil servants

Senapati Head of Military Department

Adhyakshas Superintendents

Dharmasthiyas Civil Courts

Kantakshodhanas Criminal Courts

Bandhavgarh Jail

Charaka Police lock-up

Nagarika City superintendent

Source: Vajiram and Ravi Yellow book- “Ancient India”- Chapter 6, Page 47-48 3. Which of the following is not correct with respect to Dhamma?

(a) Founded by Ashoka, Dhamma was a new religion based on Buddhism. (b) It lists out a set of principles to be practised by the people. (c) It was considered to be practical and convenient. (d) Some rock edicts mention the features of Dhamma.

Answer: (a) Explanation:

https://telegram.me/UPSCMaterials https://telegram.me/FreeUPSCMaterials https://telegram.me/MaterialforExam

Page 3: Answer Key & Exp for GS Prelim Test-5 held on 23rd …...1 Prelim IAS Test Series (2019) – GS Test 5 (23.12.2018) History 1 and Current Affairs (July 2018) Answer Key & Exp for GS

3

Prelim IAS Test Series (2019) – GS Test 5 (23.12.2018) History 1 and Current Affairs (July 2018)

Ashoka's Dhamma was a code of social ethics, and, by extension, the social and religious order that Ashoka sought to establish in the turbulent society of his time - a society that was mostly Brahmanical. Option (a) is incorrect. Although Buddhism and Hinduism may have influenced it, Ashoka’s dhamma was neither a new religion nor political philosophy. It was a way of life. Option (b) is correct. It provides a code of conduct and a set of principles to be adopted and practised by the people at large. Its concepts were so broad and humanitarian that no cultural group or religious sect could object to its propagation. Option (c) is correct. The code of ethics was both practical and convenient, as well as highly moral. Option (d) is correct. The main features of Dhamma were given on a number of rock edicts.

Major Rock Edict I: Prohibition of animal sacrifices and festivities.

Major Rock Edict II: Administration to be directed towards social welfare.

Major Rock Edict III: Liberality to Brahmins, sramanas.

Major Rock Edict IV: Consideration and non-violence to animals and courtesy to relations.

Major Rock Edict V: Appointment of dhammamahamattas entrusted with propagation of dhamma. Humane treatment of servants by masters and of prisoners by the government officials.

Major Rock Edict VI: Efficient organisation of administration.

Major Rock Edict VII: Tolerance among all the sects.

Major Rock Edict VIII: Maintenance of constant contact with rural people through the system of dhammayatras.

Major Rock Edict IX: Avoiding expensive and meaningless ceremonies and rituals.

Major Rock Edict XI: Charity, kinship, dhamma.

Major Rock Edict XII: Religious tolerance.

Major Rock Edict XlII: Change of heart after Kalinga war and replacement of berighos (sounds of war drums) by dhammaghosha (sound of peace) signifying conquest through dhamma instead of through war.

Source: Vajiram and Ravi Yellow book- “Ancient India”- Chapter 6, Page 49-50. 4. Which of the following reasons can be attributed to the rise and success of

Magadha among the mahajanpadas? (1) Broad resource base (2) Military machinery (3) Effective bureaucracy Select the correct answer using the code given below: (a) 1 and 2 only (b) 2 and 3 only (c) 1 and 3 only (d) 1, 2 and 3

Answer: (d)

https://telegram.me/UPSCMaterials https://telegram.me/FreeUPSCMaterials https://telegram.me/MaterialforExam

Page 4: Answer Key & Exp for GS Prelim Test-5 held on 23rd …...1 Prelim IAS Test Series (2019) – GS Test 5 (23.12.2018) History 1 and Current Affairs (July 2018) Answer Key & Exp for GS

4

Prelim IAS Test Series (2019) – GS Test 5 (23.12.2018) History 1 and Current Affairs (July 2018)

Explanation: Magadha was situated in south Bihar. Its capital was Rajgriha and later Patliputra. ● Statement 1 is correct - The resource base of Magadha was broad. The land was

fertile and agricultural surplus could be produced. Magadha was rich in minerals such as iron from which agricultural tools and also weapons could be produced.

● Statement 2 is correct - The forests around Magadha had many elephants and also sufficient timber to prepare war tools. Magadha had a massive military machine with vast infantry, horses, elephants, chariots and also innovative armaments such as rathamusula and mahasilakantika developed by Ajatashatru.

● Statement 3 is correct - The Nandas streamlined an administrative system with vast bureaucracy which dug canals for expansion of agriculture, promoted trade and commerce and collected taxes adding to financial muscle of Magadha.

Additional Information: ● Magadha was inhabited by the Kiratas and Magadhas. Magadha then underwent a

racial admixture with the advent of Vedic people. As it was recently Aryanised, it showed more enthusiasm than those kingdoms already under the Aryan influence. Also, Magadha was outside the pale of Vedic culture as a result of which it did not suffer from the disabilities stemming from orthodox Brahmanical culture.

Source: Vajiram and Ravi Yellow book- “Modern Indian History”- Chapter 5, Page 35-36. 5. Consider the following statements about the Indian campaign of Alexander:

(1) Alexander could not advance beyond Beas as the kingdom of Nandas was known to be very powerful.

(2) Alexander’s historians left dated records of his campaign. (3) His campaign facilitated the expansion of the Mauryan Empire in the North-

west. Which of the statements given above is/are correct? (a) 3 only (b) 2 and 3 only (c) 1, 2 and 3 (d) None of the above

Answer: (b) Explanation: The Indian campaign of Alexander the Great began in 326 BC. After conquering the Achaemenid Empire of Persia, the Macedonian king, Alexander, launched a campaign into the Indian subcontinent, part of which formed the easternmost territories of the Achaememid Empire following the Achaemenid conquest of the Indus Valley (6th century BC). The rationale for this campaign is usually said to be Alexander's desire to conquer the entire known world, which the Greeks thought ended in India. Statement 1 is incorrect. Alexander wanted to move further east of the Beas but his war-weary, diseased soldiers, due to hot climate and ten years of continuous campaigns became homesick. So, Alexander was forced to retreat. Though it is true that the Nanda

https://telegram.me/UPSCMaterials https://telegram.me/FreeUPSCMaterials https://telegram.me/MaterialforExam

Page 5: Answer Key & Exp for GS Prelim Test-5 held on 23rd …...1 Prelim IAS Test Series (2019) – GS Test 5 (23.12.2018) History 1 and Current Affairs (July 2018) Answer Key & Exp for GS

5

Prelim IAS Test Series (2019) – GS Test 5 (23.12.2018) History 1 and Current Affairs (July 2018)

empire beyond Beas was known to be powerful, but it was not the reason why Alexander could not advance. Statement 2 is correct. Alexander’s historians left valuable geographical accounts as well as clearly dated records of his campaign which enables us to build Indian chronology for subsequent events on a definite basis. They also give us important information about the prevalent social and economic conditions like sati system, fine breed of oxen and sale of girls in marketplaces. Statement 3 is correct. Alexander destroyed the power of petty states in the north-west and created a political vacuum which was filled by the expansion of the Mauryan Empire under Chandragupta Maurya. Source: Vajiram and Ravi Yellow book- “Ancient India”- Chapter 5, Page 37. 6. The Pallavas who rose to power after the Satavahanas had an extensive

administrative system. In this context consider the following statements: (1) The Pallava State was divided into Kottams. (2) Taxes on manufacturing or industries were the key sources of the State’s

revenue. (3) The Village Assemblies provided irrigation facilities to the agricultural lands. Which of the statements given above is/are correct? (a) 1 only (b) 1 and 2 only (c) 1 and 3 only (d) 2 and 3 only

Answer: (a) Explanation: The Pallava dynasty ruled in southern India whose members originated as indigenous subordinates of the Satavahanas in the Deccan, moved into Andhra, and then to Kanchi (Kanchipuram in modern Tamil Nadu state, India), where they became rulers. Their genealogy and chronology both are highly disputed. Statement 1 is correct. The Pallava State was divided into kottams which were administered by officers appointed by the king. Statement 2 is incorrect. Land tax formed the key source of the State’s revenue although Brahmadeya and Devadana lands were exempted from taxation. Manufacturing and industrial taxes did exist, but the land tax was higher. Statement 3 is incorrect. The Central government provided for irrigation facilities to the agricultural lands. The Pallava kings constructed a large number of irrigation tanks for this purpose. Source: Vajiram and Ravi Yellow book- “Medieval India”- Chapter 1, page 2-3. 7. The Ghatika in Pallava period was?

(a) A centre of higher learning (b) An irrigation technique (c) A land tax (d) A royal ritual

https://telegram.me/UPSCMaterials https://telegram.me/FreeUPSCMaterials https://telegram.me/MaterialforExam

Page 6: Answer Key & Exp for GS Prelim Test-5 held on 23rd …...1 Prelim IAS Test Series (2019) – GS Test 5 (23.12.2018) History 1 and Current Affairs (July 2018) Answer Key & Exp for GS

6

Prelim IAS Test Series (2019) – GS Test 5 (23.12.2018) History 1 and Current Affairs (July 2018)

Answer: (a) Explanation: The Pallava kings who ruled from their capital city Kanchipuram were known as patrons of scholars. The ghatikas were the most important educational institutions of this time. The most important ghatika of the Pallavas was located in Kanchipuram. Inscriptions reveal that many of the Pallava kings looked after this institution with great care. It was well-known as an important educational centre even in the 4th century A.D and an epigraph of a Kadamba king (of ancient Karnataka) records that his ancestors, a scholar named Mayura Sharman, along with his teacher Vira Sharman came to this institution from faraway Goa. The Ghatikas attracted students from all parts of India and abroad. Source: Vajiram and Ravi Yellow book- “Medieval India”- Chapter 1, page 2. 8. Consider the following statements with respect to the land grants in early Medieval

India: (1) The earliest land grants were religious. (2) Land beneficiaries were not given the power of taxation. (3) The earliest epigraphic record of land-grant in India is a Satavahana

inscription of first century BC. Which of the statements given above are correct? (a) 1 and 2 only (b) 1 and 3 only (c) 2 and 3 only (d) 1, 2 and 3

Answer: (b) Explanation: The early medieval period in Indian history has been viewed as bringing in major changes in state structure, society, economy, religion, art, scripts or in other words, in every aspect of the life of people. Land grants are said to have been a major factor behind these changes. Statement 1 is correct. The basic feature of this system was the grant of tax-free land to Brahmanas and religious institutions, and hence earliest land grants were religious in nature. However, in the post-Gupta period, even administrative officials were granted land. Statement 2 is incorrect. The beneficiaries were given the powers of taxation too, and thus this also became a reason for the disintegration of central authority in that area. Statement 3 is correct. The earliest epigraphic record of land grants in India is a Satavahana inscription of first century BC, which refers to granting a village in Ashvamedha Sacrifice. Source: Vajiram and Ravi Yellow book- “Medieval India”- Chapter 5, page 19-20. 9. Which of the following were the characteristics of early Medieval India?

(1) Unequal distribution of land and power (2) Loss of mobility of artisans

https://telegram.me/UPSCMaterials https://telegram.me/FreeUPSCMaterials https://telegram.me/MaterialforExam

Page 7: Answer Key & Exp for GS Prelim Test-5 held on 23rd …...1 Prelim IAS Test Series (2019) – GS Test 5 (23.12.2018) History 1 and Current Affairs (July 2018) Answer Key & Exp for GS

7

Prelim IAS Test Series (2019) – GS Test 5 (23.12.2018) History 1 and Current Affairs (July 2018)

(3) Centralized administration (4) Lineage based ruling Select the correct answer using the code given below: (a) 1, 2 and 3 only (b) 1, 2 and 4 only (c) 1, 3 and 4 only (d) 2, 3 and 4 only

Answer: (b) Explanation: The early medieval period of Indian history may be seen as a transitional period wherein changes in almost all institutions and spheres of Society viz. political, social, cultural as well as the economy took place. Statement 1 is correct. In the political sphere, due to the continuation as well as acceleration of land grants, the emergence of petty landed estates, the fragmentation of authority, unequal distribution of land and power and the growth of lord-vassal relationship may be noticed as emerging new trends. Statement 2 is correct. In the economic sphere, the decline of trade and commerce, the decay of urban centres, paucity and irregular and slow circulation of metallic money, the growing agrarian character of society and emergence of the self-sufficient economy due to loss of mobility of artisans were the salient features of the economy. Statement 3 is incorrect. The early medieval period has generally been perceived as marked by decentralization of polity and thus it was a decentralized administration rather than centralized administration. Statement 4 is correct. Although central authority weakened in this era ruling was largely hereditary and lineage based. Source: Vajiram and Ravi Yellow book- “Medieval India”- chapter 3, page 10-11. 10. In medieval India, the designation ‘Nad Gaundas’ was used for:

(a) Military officers (b) Village headmen (c) Hereditary revenue officials (d) Chiefs of craft guilds

Answer: (c) Explanation: In Rashtrakuta administration, Officers like nadagavundas (corrupted to nad gaundas) were in charge of smaller divisions and assisted the provincial governor. Their office was hereditary. For instance, the family of Chaunda was the nadagavunda of Mugunda-30 division for four generations from the reign of Sastha II (C 1010 AD) to the reign of Guvaladeva III (C 1125 AD) The gavundas who appear most often in inscriptions were the backbone of the medieval polity of the southern Karnataka region. As landlords and local elites, the State utilized their services to collect taxes, maintain records of landownership, bear witness to grants and transactions and even raise militia when required. Owing to strong personal

https://telegram.me/UPSCMaterials https://telegram.me/FreeUPSCMaterials https://telegram.me/MaterialforExam

Page 8: Answer Key & Exp for GS Prelim Test-5 held on 23rd …...1 Prelim IAS Test Series (2019) – GS Test 5 (23.12.2018) History 1 and Current Affairs (July 2018) Answer Key & Exp for GS

8

Prelim IAS Test Series (2019) – GS Test 5 (23.12.2018) History 1 and Current Affairs (July 2018)

allegiance to the king, they were vested with certain rights over villages. They had rights to make grants that may have occasionally been exempt from local taxes. There were two types of gavundas; the ur-gavundas who were lower in status and wielded control at the village level and the nadu-gavunda who oversaw the Nadu and were directly appointed by the king. Source: Vajiram and Ravi Yellow book- “Medieval India”- chapter 2, page 6. 11. Consider the following statements with respect to the Pala rulers who ruled from 8th

to 10th century in India: (1) Pala rulers were Buddhist and did not give patronage to other religions. (2) It was during the reign of Pala rulers that Atisa or Dipankar Srijnana visited

Tibet to purify Buddhism in that region. Which of the statements given above is/are correct? (a) 1 only (b) 2 only (c) Both 1 and 2 (d) Neither 1 nor 2

Answer: (b) Explanation: The Pala Empire was an imperial power during the Late Classical period in the Indian subcontinent, which originated in the region of Bengal. Statement 1 is incorrect. The Palas were patrons of Mahayana Buddhism. Dharmapala made the Buddhist philosopher Haribhadra his spiritual preceptor. He established the Vikramashila monastery and the Somapura Mahavihara. The Palas also supported the Saiva ascetics, typically the ones associated with the Golagi-Math. Narayana Pala himself established a temple of Shiva and was present at the place of sacrifice by his Brahmin minister. Queen of King Madanapaladeva, namely Chitramatika, made a gift of land to a Brahmin named Bateswara Swami. Hence Pala rulers were tolerant to Hinduism and the Brahmanical sect too. Statement 2 is correct. The Palas developed the Buddhist centres of learnings, such as the Vikramashila and the Nalanda universities. Nalanda, considered one of the first great universities in recorded history, reached its height under the patronage of the Palas. Noted Buddhist scholars from the Pala period included Atisa, Santaraksita, Saraha, Tilopa, Bimal Mitra, etc. Atisa Dipaṃkara Srijnana was a Buddhist Bengali religious leader and master from the Indian subcontinent. He was one of the major figures in the spread of 11th-century Mahayana and Vajrayana Buddhism in Asia and inspired Buddhist thought from Tibet to Sumatra. In 1013 CE, he travelled to the Srivijaya kingdom and stayed there for 12 years and came back to India. He is recognised as one of the greatest figures of classical Buddhism. Source: Vajiram and Ravi Yellow book- “Medieval India”- chapter 2, page 8-9. 12. Gandhiji kept many fasts for various purposes - political, social, spiritual etc during

his lifetime. Which of the following was the first political fast by Gandhiji in India?

https://telegram.me/UPSCMaterials https://telegram.me/FreeUPSCMaterials https://telegram.me/MaterialforExam

Page 9: Answer Key & Exp for GS Prelim Test-5 held on 23rd …...1 Prelim IAS Test Series (2019) – GS Test 5 (23.12.2018) History 1 and Current Affairs (July 2018) Answer Key & Exp for GS

9

Prelim IAS Test Series (2019) – GS Test 5 (23.12.2018) History 1 and Current Affairs (July 2018)

(a) Champaran Satyagraha (b) Kheda Satyagraha (c) Ahmedabad Mill Strike (d) Rowlatt Satyagraha

Answer: (c) Explanation: Option (a) is incorrect. The Champaran Satyagraha of 1917 was the first civil disobedience movement inspired by Gandhi and a major event in the Indian Independence Movement. It was a farmers' uprising that took place in Champaran district of Bihar, India during the British colonial period. The farmers were protesting the compulsion imposed on them to grow opium with barely any remuneration provided for the opium grown. Option (b) is incorrect. In 1918, crops failed in the Kheda District in Gujarat, but the government refused to remit land revenue and insisted on its collection. Gandhiji supported the peasants and advised them to withhold payment of revenue till their demand for its remission was met. The struggle was withdrawn when it was learnt that the government had issued instructions that revenue should be recovered only from those peasants who could afford to pay. It was the first non-cooperation initiative by Gandhiji in India. Option (c) is correct. In 1918, Mahatma Gandhi intervened in a dispute between the workers and mill-owners of Ahmedabad. He advised the workers to go on strike and to demand a 35 per cent increase in wages. But he insisted that the workers should not use violence against the employers during the strike. He undertook a fast unto death to strengthen the workers' resolve to continue the strike. But his fast also put pressure on the mill-owners who relented on the fourth day and agreed to give the workers a 35 per cent increase in wages. This was the first political hunger strike by Gandhi in India. Option (d) is incorrect. The Anarchical and Revolutionary Crimes Act of 1919, popularly known as the Rowlatt Act or Black Act, was a legislative act passed by the Imperial Legislative Council in Delhi on 10 March 1919, indefinitely extending the emergency measures of indefinite preventive detention, incarceration without trial and judicial review enacted in the Defense of India Act 1915 during the First World War. It was enacted in light of a perceived threat from revolutionary nationalist organizations of re-engaging in similar conspiracies as during the war. Source: Vajiram and Ravi Yellow book- “Modern Indian History”- Chapter 9, Page 184. 13. Khilafat issue arose because of the stern attitude the British adopted towards

Turkey and removal of the Khalifa from power. This was done under which treaty? (a) Treaty of Sevres (b) Treaty of Versailles (c) Treaty of Saint Germain (d) Treaty of Trianon

Answer: (a)

https://telegram.me/UPSCMaterials https://telegram.me/FreeUPSCMaterials https://telegram.me/MaterialforExam

Page 10: Answer Key & Exp for GS Prelim Test-5 held on 23rd …...1 Prelim IAS Test Series (2019) – GS Test 5 (23.12.2018) History 1 and Current Affairs (July 2018) Answer Key & Exp for GS

10

Prelim IAS Test Series (2019) – GS Test 5 (23.12.2018) History 1 and Current Affairs (July 2018)

Explanation: The Khilafat Movement (1919-1924) was an agitation by Indian Muslims allied with Indian nationalism in the years following World War I. Its purpose was to pressurise the British government to preserve the authority of the Ottoman Sultan as Caliph of Islam following the breakup of the Ottoman Empire at the end of the war. Option (a) is correct: Treaty of Sevres was a post-World War I pact between the victorious Allied powers and representatives of the government of Ottoman Turkey. The treaty abolished the Ottoman Empire and obliged Turkey to renounce all rights over Arab Asia and North Africa. Rejected by the new Turkish nationalist regime, the Treaty of Sevres was replaced by the Treaty of Lausanne in 1923. Option (b) is incorrect: Treaty of Versailles was a peace document signed at the end of World War I by the Allied and associated powers and by Germany in the Hall of Mirrors in the Palace of Versailles, France, on June 28, 1919. Option (c) is incorrect: Treaty of Saint-Germaine was a treaty concluding World War I and signed by representatives of Austria on one side and the Allied Powers on the other. Option (d) is incorrect: Treaty of Trianon was a treaty concluding World War I and signed by representatives of Hungary on one side and the Allied Powers on the other. It was signed on June 4, 1920, at the Trianon Palace at Versailles, France. Source: Vajiram and Ravi Yellow book- “Modern Indian History”- Chapter 9, Page 186. 14. “The trial immediately caught the attention of England where it even inspired the

play by Manchester theatre group highlighting the detrimental effects of industrialization and colonization.” These lines are related to which incident? (a) Kanpur conspiracy case (b) Meerut conspiracy case (c) Kakori conspiracy case (d) Chittagong armoury raid

Answer: (b) Explanation: Option (a) is incorrect: Kanpur conspiracy case: One of the most prominent cases in the British period relating to conspiracy to wage war was that of Manabendra Nath Roy Vs. Emperor. The other accused were Nalini Bhushan Das Gupta, Mohd. Shaukat Usmani, Muzaffar Ahmad and Sripat Amrit Dange. All were charged under Section 121-A, i.e. conspiracy to wage war and were sentenced to four years of rigorous imprisonment on 20th May 1924. Roy, the principal accused, could not be arrested as he was away in Europe. He was one of the important members of Presidium of the Communist International. This case signifies the initial growth period of Marxism in India and role of Kanpur in it. It is no surprise then, that the following year in 1925 the 1st All India Communist Conference was held at Kanpur. Option (b) is correct: The Meerut Conspiracy Case was a controversial court case initiated in British India in March 1929 and decided in 1933. Several trade unionists, including three Englishmen, were arrested for organizing an Indian railway strike. The British Government convicted 33 leftist trade union leaders under a false lawsuit. The trial

https://telegram.me/UPSCMaterials https://telegram.me/FreeUPSCMaterials https://telegram.me/MaterialforExam

Page 11: Answer Key & Exp for GS Prelim Test-5 held on 23rd …...1 Prelim IAS Test Series (2019) – GS Test 5 (23.12.2018) History 1 and Current Affairs (July 2018) Answer Key & Exp for GS

11

Prelim IAS Test Series (2019) – GS Test 5 (23.12.2018) History 1 and Current Affairs (July 2018)

immediately caught attention in England, where it inspired the 1932 play Meerut by Manchester street theatre group the 'Red Megaphones', highlighting the detrimental effects of colonisation and industrialization. Option (c) is incorrect: Kakori Conspiracy Case (1925 AD)- Also called the Kakori train robbery or Kakori Case it was a train robbery against the British Indian Government. It was organized by revolutionary organisation, i.e., Hindustan Republican Association (HRA) under the leadership of Ram Prasad Bismil and supported by Ashfaqulla Khan, Rajendra Lahiri, Chandrashekhar Azad, Sachindra Bakshi, Keshab Chakravarty, Manmathnath Gupta and Banwari Lal. Option (d) is incorrect: The Chittagong armoury raid, also known as the Chittagong uprising, was an attempt on 18 April 1930 to raid the armoury of police and auxiliary forces from the Chittagong armoury in the Bengal Presidency of British India (now in Bangladesh) by armed Indian independence fighters led by Surya Sen. Source: Vajiram and Ravi Yellow book- “Modern Indian History”- Chapter 9, Page 194. 15. Which of the following were the recommendations of the Simon Commission?

(1) The Provinces should be given full autonomy. (2) The Governor should have discretionary power related to internal security. (3) The number of members of the provincial legislative council should be

decreased. (4) The Government of India should have complete control over the high court. Select the correct answer using the code given below: (a) 1, 2 and 3 only (b) 1, 2 and 4 only (c) 1, 3 and 4 only (d) 2, 3 and 4 only

Answer: (b) Explanation: The Indian Statutory Commission, commonly referred to as the Simon Commission, was a group of seven British Members of Parliament under the chairmanship of Sir John Allsebrook Simon. The Commission arrived in British India in 1928 to study constitutional reform in Britain's most important colonial dependency. The recommendations of the Commission were as follows: There should be constitutional reconstruction. It would be a federal constitution. The provinces should be given full autonomy including law. Thus, Statement 1 is correct The Governor should have discretionary power to relate to internal securities and administrative power to protect the different communities. Thus, Statement 2 is correct. The number of members of provincial legislative councils should be increased. Nowhere it was stated to decrease the number of members of the Provincial Legislative Council. Thus, Statement 3 is incorrect. The Government of India should have complete control over the high court. Thus, Statement 4 is correct.

https://telegram.me/UPSCMaterials https://telegram.me/FreeUPSCMaterials https://telegram.me/MaterialforExam

Page 12: Answer Key & Exp for GS Prelim Test-5 held on 23rd …...1 Prelim IAS Test Series (2019) – GS Test 5 (23.12.2018) History 1 and Current Affairs (July 2018) Answer Key & Exp for GS

12

Prelim IAS Test Series (2019) – GS Test 5 (23.12.2018) History 1 and Current Affairs (July 2018)

Source: Vajiram and Ravi Yellow book- “Modern Indian History”- Chapter 9, Page 200. 16. Which of the following were the reasons for the introduction of the Ryotwari system

in the Madras Presidency in 1820 by Thomas Munroe? (1) Rising agricultural commodity prices (2) Aiming for enhanced income (3) Economic theory of David Ricardo prevalent at that time Select the correct answer using the code given below: (a) 1 and 2 only (b) 2 and 3 only (c) 1, 2 and 3 (d) None of the above

Answer: (c) Explanation: The Ryotwari system was a land tenure system in British India, introduced by Sir Thomas Munro in 1820 based on system administered by Captain Alexander Reed in the Baramahal district. It allowed the government to deal directly with the peasant (ryot) for revenue collection, and gave the peasant freedom to give up or acquire new land for cultivation. Statement 1 is correct: After 1810 agricultural commodity prices rose increasing the value of the harvest produced, so there was the possibility of greater rent. Statement 2 is correct: On analyzing the income from Bengal ,i.e. the revenue demand and the permanent settlement, the colonial state could not claim any share in the enhanced income. Statement 3 is correct: The English devised policies according to the income theories they were familiar with. The prevalent economic theory during that time was that of David Ricardo which stated that landowner should have a claim only to the average rent that prevailed at a given time. When the land yielded more than this average rent, the landowner had a surplus that the state needed to tax. If the tax was not levied, cultivators were likely to turn into renters, and their surplus income was unlikely to be productively invested in the improvement of the land. Source: Vajiram and Ravi Yellow book- “Modern Indian History”- Chapter 3, Page 41. 17. Arrange the following States in the sequence of their annexation (from earliest to

latest) under the Doctrine of lapse introduced by Lord Dalhousie: (1) Jaitpur (2) Satara (3) Baghat (4) Sambalpur Which of the following is the correct order? (a) 1-2-3-4 (b) 1-3-2-4

https://telegram.me/UPSCMaterials https://telegram.me/FreeUPSCMaterials https://telegram.me/MaterialforExam

Page 13: Answer Key & Exp for GS Prelim Test-5 held on 23rd …...1 Prelim IAS Test Series (2019) – GS Test 5 (23.12.2018) History 1 and Current Affairs (July 2018) Answer Key & Exp for GS

13

Prelim IAS Test Series (2019) – GS Test 5 (23.12.2018) History 1 and Current Affairs (July 2018)

(c) 2-1-3-4 (d) 2-1-4-3

Answer: (d) Explanation: The correct sequence is: Satara>Jaitpur> Sambalpur>Baghat The Doctrine of lapse was an annexation policy applied by Lord Dalhousie in India before 1858. This doctrine was based on the idea that in case the ruler of a dependent state died childless, the right of ruling over the State reverted or ‘lapsed’ to the sovereign. This position, however, was complicated by adoption. Hindus, including Hindu rulers, attached great importance to the performance of their funeral rites by their sons and if they died without a male issue, they resorted to adoption. An adopted son was allowed to inherit private property, but the political rights of ruling a State were different. This involved the welfare of millions of people. Before 1818 A.D., the East India Company was not sovereign over India and did not interfere in matters of succession of other Indian States. The Company agreed to adoptions in States dependent upon itself as a matter of course during 1818 to 1834 A.D. In 1834 A.D. however, the Company came to a resolution that adoption was permissible as a matter of grace and not right and that previous permission of the Company was required for this. By 1841 A.D., the Company had resolved that adoption was not to be granted as a rule because all opportunities for the extension of British territory were to be welcomed. By applying the Doctrine of lapse, Dalhousie annexed the States of Satara (1848 A.D.), Jaitpur (1848 A.D.), Sambalpur (1849 A.D.), Baghat (1850 A.D.), Jhansi (1855 A.D.), and Nagpur (1854 A.D.). Annexation by lapse of ‘Karauli’ was disallowed by the Court of Directors. Source: Vajiram and Ravi Yellow book- “Modern Indian History”- Chapter 3, Page 52. 18. With reference to Raja Ram Mohan Roy, consider the following statements:

(1) He considered different religions as national embodiments of universal theism. (2) He defended the polytheism of Hinduism and trinitarianism of Christianity. (3) He considered departure from the scriptures as justified, if reason demanded

it. Which of the statements given above is/are correct? (a) 1 only (b) 1 and 3 only (c) 2 and 3 only (d) 1, 2 and 3

Answer: (b) Explanation: Raja Ram Mohan Roy (1772-1833) has been considered as ‘father of Indian Renaissance’. He believed in the modern scientific approach and principles of human dignity and social equality.

https://telegram.me/UPSCMaterials https://telegram.me/FreeUPSCMaterials https://telegram.me/MaterialforExam

Page 14: Answer Key & Exp for GS Prelim Test-5 held on 23rd …...1 Prelim IAS Test Series (2019) – GS Test 5 (23.12.2018) History 1 and Current Affairs (July 2018) Answer Key & Exp for GS

14

Prelim IAS Test Series (2019) – GS Test 5 (23.12.2018) History 1 and Current Affairs (July 2018)

Statement 1 is correct: He considered different religions as national embodiments of universal theism. Statement 2 is incorrect: He defended the basic and universal principles of all religions - such as the monotheism of the Vedas and Unitarianism of Christianity - while attacking the polytheism of Hinduism and trinitarianism of Christianity. Statement 3 is correct: Through the platform of Atmiya Sabha (set up in 1814), he propagated the monotheistic ideas of the Vedanta and to campaign against idolatry, caste rigidities, meaningless rituals and other social ills. Strongly influenced by rationalist ideas, he declared that Vedanta is based on reason and that, if reason demanded it, even a departure from the scriptures is justified. Source: Spectrum’s “A brief history of Modern India”- Chapter 8, page no. 213-214, Chapter 9, page no. 228. 19. Which of the following is/are associated with Dr B. R. Ambedkar?

(1) Mahad Satyagraha (2) Mook Nayak (3) Harijan Sevak Sangh Select the correct answer using the code given below: (a) 1 only (b) 1 and 2 only (c) 2 and 3 only (d) 1, 2 and 3

Answer: (b) Explanation: Dr Bhimrao Ambedkar (1891-1956) was an Indian jurist, economist, politician and social reformer who inspired the Dalit Buddhist movement and campaigned against social discrimination towards the untouchables (Dalits), while also supporting the rights of women and labour. Statement 1 is correct: He led the Mahad Satyagraha in March 1927 to challenge the regressive customs of the caste Hindus. He stressed the necessity of removing ideas of ‘high’ and ‘low’ and inculcating self-elevation through self-help, self-respect and self-knowledge. He led a procession of some 2,500 ‘untouchables’ through the town of Mahad to the Chawdar tank, a public source of water tank from which the untouchables were not allowed to draw water. He took water from the tank and drank it. There were huge protests by caste Hindus. Statement 2 is correct: Dr B.R. Ambedkar started the Marathi weekly ‘Mook Nayak’, or the Mute Hero, to give voice to untouchables in the society and share their problems in 1920. Statement 3 is incorrect: Harijan Sevak Sangh was founded by Mahatma Gandhi in 1932 to eradicate untouchability in India. Dr Ambedkar established the Bahishkrit Hitakarini Sabha in 1924 to highlight the difficulties and grievances of the Dalits before the government. Its motto was: ‘Educate, Agitate and Organise’.

https://telegram.me/UPSCMaterials https://telegram.me/FreeUPSCMaterials https://telegram.me/MaterialforExam

Page 15: Answer Key & Exp for GS Prelim Test-5 held on 23rd …...1 Prelim IAS Test Series (2019) – GS Test 5 (23.12.2018) History 1 and Current Affairs (July 2018) Answer Key & Exp for GS

15

Prelim IAS Test Series (2019) – GS Test 5 (23.12.2018) History 1 and Current Affairs (July 2018)

Source: Spectrum’s “A brief history of Modern India”- Chapter 8, page no. 224-225 and Vajiram and Ravi Yellow book- “Modern Indian History”- Chapter 4, page no. 72. 20. Consider the following statements:

(1) Ramakrishna Paramahamsa laid the foundations of the Ramakrishna Math. (2) Ramakrishna Paramahamsa recognised the fundamental oneness of all

religions. (3) Swami Vivekanand advocated for simple living without using technology and

modern science. Which of the statements given above are correct? (a) 1 and 2 only (b) 2 and 3 only (c) 1 and 3 only (d) 1, 2 and 3

Answer: (a) Explanation: Ramakrishna Paramahamsa (1836-1886) was a poor priest at the Kali temple in Dakshineshwar, on the outskirts of Calcutta, who believed in the message of Vedanta. Against the onslaught of westernisation and modernisation, Ramakrishna sought to attain salvation through devotion, meditation and renunciation. Two objectives of the Ramakrishna movement were: (i) to bring into existence a band of monks dedicated to a life of renunciation and

practical spirituality, from among whom teachers and workers would be sent out to spread the universal message of Vedanta as illustrated in the life of Ramakrishna, and

(ii) in conjunction with lay disciples to carry on preaching, philanthropic and charitable works, looking upon all men, women and children, irrespective of caste, creed or colour, as veritable manifestations of the Divine.

Statement 1 is correct: Ramakrishna Paramahamsa himself laid the foundations of the Ramakrishna Math with his young monastic disciples as a nucleus to fulfil the first objective. Statement 2 is correct: He recognised the fundamental oneness of all religions and emphasised that Krishna, Hari, Ram, Christ, Allah are different names for the same God and that there are many ways to God and salvation. Swami Vivekananda: The second objective was taken up by Swami Vivekananda after Ramakrishna’s death when he founded the Ramakrishna Mission in 1897. He spread Ramakrishna’s message and tried to reconcile it to the needs of contemporary Indian society. He emerged as the preacher of neo-Hinduism. His mission was to bridge the gulf between paramartha (service) and vyavahara (behaviour), and between spirituality and day-to-day life. He pointed out that the masses needed two kinds of knowledge—secular knowledge about how to work for their economic uplift and the spiritual knowledge to have faith in themselves and strengthen their moral sense.

https://telegram.me/UPSCMaterials https://telegram.me/FreeUPSCMaterials https://telegram.me/MaterialforExam

Page 16: Answer Key & Exp for GS Prelim Test-5 held on 23rd …...1 Prelim IAS Test Series (2019) – GS Test 5 (23.12.2018) History 1 and Current Affairs (July 2018) Answer Key & Exp for GS

16

Prelim IAS Test Series (2019) – GS Test 5 (23.12.2018) History 1 and Current Affairs (July 2018)

Statement 3 is incorrect: Vivekanand advocated for using technology and modern science in the service of humanity. Source: Spectrum’s “A brief history of Modern India”- Chapter 9, page no. 240-243. 21. Match List-I with List-II and select the correct answer using the code given below:

List-I (Reform Movement in India)

List-II (Leaders)

A Deoband Movement 1 Dadu Mian

B Wahabi Movement 2 Mirza Ghulam Ahmed

C Faraizi Movement 3 Mohammad Qasim Nanotavi

D Ahmadiyya Movement 4 Syed Ahmed Barelvi

Codes A B C D (a) 2 4 3 1 (b) 1 4 3 2 (c) 3 4 1 2 (d) 1 3 4 2

Answer: (c) Explanation: Deoband Movement (Darul Uloom) - The orthodox section organised this movement among the Muslim ulema as a revivalist movement with the twin objectives of propagating pure teachings of the Quran and Hadis among Muslims and keeping alive the spirit of jihad against the foreign rulers. It was begun in Deoband, Saharanpur district in 1866 by Mohammad Qasim Nanotavi (1832-80) and Rashid Ahmed Gangohi (1828-1905) to train religious leaders for the Muslim community. Wahabi Movement: The teachings of Abdul Wahab of Arabia and the preachings of Shah Walliullah (1702-1763) inspired this essentially revivalist response to Western influences and the degeneration which had set in among Indian Muslims and called for a return to the true spirit of Islam. Shah Walliullah tried to organise Muslims around the two-fold ideals of this movement: the desirability of harmony among the four schools of Muslim jurisprudence which had

divided the Indian Muslims (he sought to integrate the best elements of the four schools);

Recognition of the role of individual conscience in a religion where conflicting interpretations were derived from the Quran and the Hadis.

Other leaders like Shah Abdul Aziz and Syed Ahmed Barelvi further popularised the teachings of Walliullah and his movement. Initially, the movement was directed against the Sikhs in Punjab, but after the British annexation of Punjab (1849), the movement was directed against the British. During the 1857 Revolt, the Wahabis’ played an important role in spreading anti-British feelings. The Wahabi Movement fizzled out in the face of British military might in the 1870s.

https://telegram.me/UPSCMaterials https://telegram.me/FreeUPSCMaterials https://telegram.me/MaterialforExam

Page 17: Answer Key & Exp for GS Prelim Test-5 held on 23rd …...1 Prelim IAS Test Series (2019) – GS Test 5 (23.12.2018) History 1 and Current Affairs (July 2018) Answer Key & Exp for GS

17

Prelim IAS Test Series (2019) – GS Test 5 (23.12.2018) History 1 and Current Affairs (July 2018)

Faraizi Movement: This movement founded by Haji Shariatullah in 1818, emphasized the Islamic pillars of faith. It was started with the aim of eradication of social innovations or un-Islamic practices current among the Muslims of the East Bengal region and draw their attention to their duties as Muslims. Later under the leadership of Haji’s son, Dadu Mian, the movement became revolutionary from 1840 onwards. He gave the movement an organisational system from the village to the provincial level with a Khalifa or authorised deputy at every level. They fought against the Hindu landlords and even police using clubs and organised themselves into a paramilitary force. After several arrests, Dadu Mian died in 1862, which ended the movement. Ahmadiyya Movement: The Ahmadiyya forms a sect of Islam which originated from India. Mirza Ghulam Ahmad founded it in 1889. It was based on liberal principles. It described itself as the standard-bearer of Mohammedan Renaissance, and based itself, like the Brahmo Samaj, on the principles of universal religion of all humanity, opposing jihad (sacred war against non-Muslims). Hence, option (c) is the correct answer. Source: Vajiram and Ravi Yellow book- “Modern Indian History”- Chapter 4, page no. 68-69. 22. Consider the following statements:

(1) One of the objectives of the Singh Sabha Movement was to make available western education to the Sikhs.

(2) The Akali Movement was a communal movement, aimed at liberating Gurudwaras from the control of corrupt Mahants.

Which of the statements given above is/are correct? (a) 1 only (b) 2 only (c) Both 1 and 2 (d) Neither 1 nor 2

Answer: (a) Explanation: The Singh Sabha Movement was founded at Amritsar in 1873 with a two-fold objective: (i) to make available modern western education to the Sikhs, (thus, Statement 1 is

correct) (ii) to counter the proselytising activities of Christian missionaries as well as the Brahmo

Samajists, Arya Samajists and Muslim maulvis. For the first objective, a network of Khalsa schools was established by the Sabha throughout Punjab. For the second objective, everything that went against the Gurus’ teachings was rejected, and rites and customs considered to be consistent with Sikh doctrine were sought to be established. Statement 2 is incorrect: The Akali movement (also known as Gurudwara Reform Movement) was an offshoot of the Singh Sabha Movement. It aimed at liberating the Sikh gurudwaras from the control of corrupt Udasi mahants (the post had become hereditary). These mahants were a loyalist and reactionary lot, enjoying government patronage. The Akali Movement was a regional movement but not having a communal tone. The

https://telegram.me/UPSCMaterials https://telegram.me/FreeUPSCMaterials https://telegram.me/MaterialforExam

Page 18: Answer Key & Exp for GS Prelim Test-5 held on 23rd …...1 Prelim IAS Test Series (2019) – GS Test 5 (23.12.2018) History 1 and Current Affairs (July 2018) Answer Key & Exp for GS

18

Prelim IAS Test Series (2019) – GS Test 5 (23.12.2018) History 1 and Current Affairs (July 2018)

government tried its repressive policies against the non-violent non-cooperation satyagraha launched by the Akalis in 1921, but had to bow before popular demands; it passed the Sikh Gurudwaras Act in 1922 (amended in 1925) which gave the control of gurudwaras to the Sikh masses to be administered through Shiromani Gurudwara Prabandhak Committee (SGPC) as the apex body. Source: Vajiram and Ravi Yellow book- “Modern Indian History”- Chapter 4, page no. 69. 23. Consider the following statements:

Assertion (A): The reform movements of nineteenth-century India aroused the communal consciousness among the Indians. Reason (R): These movements had a narrow social base viz. educated and urban middle classes. Select the correct answer using the code given below: (a) Both (A) and (R) are true and (R) is the correct explanation of (A). (b) Both (A) and (R) are true but (R) is not the correct explanation of (A). (c) (A) is true but (R) is wrong. (d) (A) is wrong but (R) is true.

Answer: (b) Explanation: Assertion statement is correct. The Hindu reformers had a negative opinion of middle ages and praised only the ancient times, on the other hand, Muslim reformers turned towards the West Asian history to seek their moments of pride. This raised the communal consciousness. Reason statement is correct. The movement had a very narrow social base viz. urban and educated middle class. The needs of vast masses which were peasants and urban poor were ignored. In its effort to reform religion and social ills, these organizations encouraged pseudo-scientific thinking and mysticism. Many parts of Indian culture were ignored such as art, architecture, literature, music, science and technology. Therefore, both (A) and (R) are true but (R) is not the correct explanation of (A). Additional Information: The reform movements of nineteenth-century India helped in:

Liberating individual from the clinch of fear and uncritical submission to exploitation by the religious leaders/priests.

The human capacity to think and reason was emphasised upon by reformers.

The reformers were able to uplift the morale of people by opposing the idea that Indian religion and society was inferior.

It mitigated the sense of humiliation of people, especially the middle class.

The reformers aimed at modernisation rather than a blind following of an alien culture.

The movements ended the cultural and intellectual isolation of Indians. Source: Vajiram and Ravi Yellow book- “Modern Indian History”- Chapter 4, page no. 74-75.

https://telegram.me/UPSCMaterials https://telegram.me/FreeUPSCMaterials https://telegram.me/MaterialforExam

Page 19: Answer Key & Exp for GS Prelim Test-5 held on 23rd …...1 Prelim IAS Test Series (2019) – GS Test 5 (23.12.2018) History 1 and Current Affairs (July 2018) Answer Key & Exp for GS

19

Prelim IAS Test Series (2019) – GS Test 5 (23.12.2018) History 1 and Current Affairs (July 2018)

24. With respect to the communist strands in Indian national movement, match the following pairs:

Communist Leader Association/ Organisation

A Indian Socialist Labour Party 1 M.N. Roy

B Labour Kisan Party 2 S.A. Dange

C Labour Swaraj Party 3 S. Chettiar

D Communist Party of India 4 Muzaffar Ahmad

Select the correct answer using the code given below:

A B C D (a) 2 1 4 3 (b) 4 3 2 1 (c) 3 2 1 4 (d) 2 3 4 1

Answer: (d) Explanation: Communism and Indian National Movement:

Russian revolution and disenchantment with the withdrawal of non-cooperation movement prepared the grounds for communism in India.

Working class militancy was on increase plus the economic conditions were deteriorating.

M.N Roy formed the first Communist Party of India in Tashkent in 1920.

S.A Dange launched the Indian Socialist Labour Party as a wing of Congress.

S.Chettiar announced the formation of Labour Kisan Party in Madras,

Muzaffar Ahmad formed the Labour Swaraj Party with the help of Kazi Nazrul Islam. Hence the correct choice is option (d). Source: Vajiram and Ravi Yellow book- “Modern Indian History”- Chapter 11, page no. 264-265. 25. Netaji Subhash Chandra Bose was instrumental in involving women in military

action. In this context, consider the following statements: (1) He raised Congress women’s volunteer corps under the leadership of Captain

Lakshmi Sehgal. (2) In the Indian National Army, he added a women’s regiment called the Rani of

Jhansi regiment. Which of the statements given above is/are correct? (a) 1 only (b) 2 only (c) Both 1 and 2 (d) Neither 1 nor 2

https://telegram.me/UPSCMaterials https://telegram.me/FreeUPSCMaterials https://telegram.me/MaterialforExam

Page 20: Answer Key & Exp for GS Prelim Test-5 held on 23rd …...1 Prelim IAS Test Series (2019) – GS Test 5 (23.12.2018) History 1 and Current Affairs (July 2018) Answer Key & Exp for GS

20

Prelim IAS Test Series (2019) – GS Test 5 (23.12.2018) History 1 and Current Affairs (July 2018)

Answer: (b) Explanation: Netaji Subhash Chandra Bose has been intricately connected with women movement in India:

Statement 1 is not correct. Netaji was instrumental in raising Congress women’s volunteer corps under the leadership of Colonel Latika Ghosh.

Statement 2 is correct. A women’s regiment was added to the expatriate army in south-east Asia which was called Rani of Jhansi Regiment.

In 1943, training camps were organised which were joined by 1500 women from both elite and working class.

Source: Vajiram and Ravi Yellow book- “Modern Indian History”- Chapter 11, page no. 288. 26. With respect to the rise of communism in India, identify the leader from the following

statements: (1) He published a book titled ‘Gandhi vs Lenin’. (2) He was the editor of a communist journal titled ‘The Socialist’. (3) He was one of the 4 communists tried under Kanpur Conspiracy Case. Which of the leader fits the description given above? (a) Muzaffar Ahmad (b) Shaukat Usmani (c) S.A. Dange (d) M.N. Roy

Answer: (c) Explanation: Among all the communist leaders in India, S.A Dange holds a glorious position.

He organised a communist group in Bombay.

For participating in Non-Cooperation Movement, he discontinued his studies.

In 1921, he published a book titled ‘Gandhi vs Lenin’ in which he showed his preference for socialism.

In 1922, he edited the communist journal ‘The Socialist’.

In this journal, he announced the formation of Indian Socialist Labour Party of Indian National Congress,

Source: Vajiram and Ravi Yellow book- “Modern Indian History”- Chapter 11, page no. 264. 27. Consider the following statements in the context of the origin of two-nation theory:

(1) Poet Iqbal was the first one to propose the idea of a separate Muslim nation. (2) Rehmat Ali coined the term Pakistan. (3) Mohammed Ali Jinnah recommended the alliance of three independent

Muslim nations – Northwest, North East and South (Hyderabad region). Which of the statements given above is/are correct? (a) 2 and 3 only (b) 2 only

https://telegram.me/UPSCMaterials https://telegram.me/FreeUPSCMaterials https://telegram.me/MaterialforExam

Page 21: Answer Key & Exp for GS Prelim Test-5 held on 23rd …...1 Prelim IAS Test Series (2019) – GS Test 5 (23.12.2018) History 1 and Current Affairs (July 2018) Answer Key & Exp for GS

21

Prelim IAS Test Series (2019) – GS Test 5 (23.12.2018) History 1 and Current Affairs (July 2018)

(c) 1, 2 and 3 (d) None of the above

Answer: (b) Explanation: Evolution of two-nation theory was a gradual phenomenon shaping up with the contributions from different Muslim leaders. Statement 1 is incorrect. In the early 1930s this idea of a separate Muslim nation was proposed by a young Muslim intellectual Rehmat Ali and later developed further by poet Iqbal. Statement 2 is correct. Later Rehmat Ali published a leaflet called ‘Now or Never’ in which he coined the term Pakistan. Statement 3 is incorrect. Rehmat Ali recommended three independent Muslim nations allying with the North West, North East and South. Source: Vajiram and Ravi Yellow book- “Modern Indian History”- Chapter 11, page no. 262. 28. 'All India States Peoples’ Conference' set up in 1927 marked the consolidation of

movements of people in princely states. Which of the following leader(s) was/were the prime mover/movers behind this organization? (1) Maniklal Kothari (2) Balwant Rai Mehta (3) G.R. Abhyankar Select the correct answer using the code given below: (a) 1 only (b) 1 and 3 only (c) 2 and 3 only (d) 1, 2 and 3

Answer: (d) Explanation: The All India States Peoples' Conference (AISPC), founded in Bombay as its headquarters in 1927, had Maniklal Kothari, Balwant Rai Mehta, G. R. Abhyankar as its prime movers. It was a conglomeration of political movements in the princely states of the British Raj. It was established to encourage political dialogue between the princely class of India, and the British Raj, upon the issues of governance, political stability and future of India. The body had no popular representation until the 1930s, when it opened up its ranks to membership from across the political spectrum. Jawahar Lal Nehru, who would become the first Prime Minister of India in 1947, was invited to become the President of the All-India body in 1935, became the President in 1939 and remained so until 1946. Hence option (d) is the correct alternative. Source: Vajiram and Ravi Yellow book- “Modern Indian History”- Chapter 11, page no. 276.

https://telegram.me/UPSCMaterials https://telegram.me/FreeUPSCMaterials https://telegram.me/MaterialforExam

Page 22: Answer Key & Exp for GS Prelim Test-5 held on 23rd …...1 Prelim IAS Test Series (2019) – GS Test 5 (23.12.2018) History 1 and Current Affairs (July 2018) Answer Key & Exp for GS

22

Prelim IAS Test Series (2019) – GS Test 5 (23.12.2018) History 1 and Current Affairs (July 2018)

29. With respect to the struggle of people in princely states, arrange the following events in chronological order (earliest to latest): (1) Sir Harcourt Butler Committee submitted its report accepting people’s say in

their governance. (2) Indian National Congress passed a resolution asking princes to introduce

responsible governments. (3) Nehru Report pleaded for All India Federation comprising of both British India

and princely states. Select the correct answer using the code given below: (a) 1-2-3 (b) 2-1-3 (c) 2-3-1 (d) 3-2-1

Answer: (c) Explanation: All India States Peoples’ Conference became the main vehicle for carrying on the political struggle in princely states. Indian National Congress passed a resolution asking princes to introduce responsible governments in 1923. Nehru Report pleaded for All India Federation comprising of both British India and Princely states in 1928. Sir Harcourt Butler committee submitted its report accepting people’s say in their governance in 1929. Hence the correct choice is (c). Source: Vajiram and Ravi Yellow book- “Modern Indian History”- Chapter 11, page no. 276. 30. Leaders of the Indian National Congress came in touch with international ideologies

and participated in conferences outside India. Consider the following statements in this regard: (1) Dadabhai Naoroji attended the International Congress held in Brussels

against Colonial Oppression and Imperialism. (2) Jawahar Lal Nehru attended the International Socialist Conference in

Amsterdam. Which of the statements given above is/are correct? (a) 1 only (b) 2 only (c) Both 1 and 2 (d) Neither 1 nor 2

Answer: (d) Explanation: Leaders of the INC were aware of international trends and interacted with influencers outside.

https://telegram.me/UPSCMaterials https://telegram.me/FreeUPSCMaterials https://telegram.me/MaterialforExam

Page 23: Answer Key & Exp for GS Prelim Test-5 held on 23rd …...1 Prelim IAS Test Series (2019) – GS Test 5 (23.12.2018) History 1 and Current Affairs (July 2018) Answer Key & Exp for GS

23

Prelim IAS Test Series (2019) – GS Test 5 (23.12.2018) History 1 and Current Affairs (July 2018)

Statement 1 is incorrect. Dadabhai Naoroji attended the International Socialist Conference in Amsterdam. Statement 2 is incorrect. Jawahar Lal Nehru attended the International Congress against Colonial Oppression and Imperialism held in Brussels. Hence the correct answer option is (d). Source: Vajiram and Ravi Yellow book- “Modern Indian History”- Chapter 11, page no. 278-279. 31. In August 1928, Jawahar Lal Nehru launched the Independence for India League.

Which among the following was not an objective of this organisation? (a) To counter the concept of dominion status (b) To plead for complete independence from the British (c) To work for the establishment of an Indian Republic on socialist lines (d) All of the above were the objectives of the Independence for India League

Answer: (d) Explanation: In November 1927, Nehru visited the Soviet Union on an official invitation. Greatly impressed by the novel experiments he desired to replicate it in the Indian context. In 1928, he launched the Independence for India league with Subhash Chandra Bose as Secretary and S. Srinivasa Iyengar as President. The objectives of the organisation were:

Option (a) is correct: To counter the concept of dominion status

Option (b) is correct: To plead for complete independence from the British

Option (c) is correct: To work for the establishment of an Indian Republic on socialist lines

Hence the correct answer is (d). Source: Vajiram and Ravi Yellow book- “Modern Indian History”- Chapter 11, page no. 280. 32. The formation of Congress Socialist Party marked an important event for

establishing a socialistic trend in Indian polity. Which of the following events led to its formation? (1) Withdrawal of Non-Cooperation Movement disillusioned the youth leaders. (2) The dialogue in Nashik jail built a consensus for the formation of the party. Select the correct answer using the code given below: (a) 1 only (b) 2 only (c) Both 1 and 2 (d) Neither 1 nor 2

Answer: (b) Explanation: Some Indian nationalists became actively involved in the International Communist movement and the Communist Party of India was established in 1925. Factors that led to its formation:

https://telegram.me/UPSCMaterials https://telegram.me/FreeUPSCMaterials https://telegram.me/MaterialforExam

Page 24: Answer Key & Exp for GS Prelim Test-5 held on 23rd …...1 Prelim IAS Test Series (2019) – GS Test 5 (23.12.2018) History 1 and Current Affairs (July 2018) Answer Key & Exp for GS

24

Prelim IAS Test Series (2019) – GS Test 5 (23.12.2018) History 1 and Current Affairs (July 2018)

Statement 1 is incorrect: Failure of civil disobedience movement led to establishment of Congress Socialist Party and not the failure of Non-Cooperation Movement.

Statement 2 is correct: Consensus of left-wing leaders at Nashik jail. After the Civil Disobedience Movement some of the leaders were lodged in the Nashik jail. These leaders discussed amongst themselves about the possibility of a forum or party based on socialist principles. Some of the leaders involved in the Nasik dialogue were Asoka Mehta, Achyut Patwardhan, Jaiprakash Narayan, N G Goray, Minoo Masani and Yusuf Meherally. Source: Vajiram and Ravi Yellow book- “Modern Indian History”- Chapter 11, page no. 281. 33. Mahatma Gandhi made 'untouchability' as an issue of public concern for the first

time in 1920s. Which of the following effort(s) is/are correct in this regard? (1) The Non-Cooperation Resolution, 1920 mentioned the removal of

untouchability as a necessary precondition for attaining Swaraj. (2) When the Communal Award in 1932 was offered, Gandhi went on a fast unto

death. (3) All India Depressed Classes League was formed in 1935 with Mahatma

Gandhi as its President. Select the correct answer using the code given below: (a) 3 only (b) 1 and 2 only (c) 1 and 3 only (d) 1, 2 and 3

Answer: (b) Explanation: Gandhi and untouchability:

Statement 1 is correct. The 1920 Non-Cooperation resolution mentioned the removal of untouchability as a necessary precondition for attaining Swaraj.

Statement 2 is correct. When the Communal Award was announced in 1932, Gandhi went on a fast unto death. Hence he set up the Harijan Sevak Sangh and launched his weekly ‘Harijan’.

Statement 3 is incorrect. All India Depressed Classes League was formed in 1935 with Jagjivan Ram as its President.

Source: Vajiram and Ravi Yellow book- “Modern Indian History”- Chapter 11, page no. 273. 34. Which of the following statements are correct about B.R. Ambedkar?

(1) He presided over the first Conference of All India Depressed Classes at Nagpur.

(2) He set up the All India Depressed Classes Congress. (3) He burnt a copy of Manusmriti, the text which authorized untouchability. (4) He suggested a complete overhaul of Hindu theology and Hindu society.

https://telegram.me/UPSCMaterials https://telegram.me/FreeUPSCMaterials https://telegram.me/MaterialforExam

Page 25: Answer Key & Exp for GS Prelim Test-5 held on 23rd …...1 Prelim IAS Test Series (2019) – GS Test 5 (23.12.2018) History 1 and Current Affairs (July 2018) Answer Key & Exp for GS

25

Prelim IAS Test Series (2019) – GS Test 5 (23.12.2018) History 1 and Current Affairs (July 2018)

Select the correct answer using the code given below: (a) 1, 2 and 3 only (b) 2 and 3 only (c) 1 and 4 only (d) 2, 3 and 4 only

Answer: (d) Explanation: Dr B.R Ambedkar, a scholar, a social reformer and a leader, dedicated his life to eradicating social inequality in India. Statement 1 is incorrect. Before Ambedkar’s efforts, a Conference of All India Depressed Classes was held at Nagpur which was presided by Raja of Kolhapur. B.R Ambedkar was chosen as the Vice-President of All India Depressed Classes Association. Statement 2 is correct. Later he formed his association called All India Depressed Classes Congress. Statement 3 is correct. He burnt a copy of Manusmriti, the text which authorized untouchability. Statement 4 is correct. He suggested a complete overhaul of Hindu theology and Hindu society as both promoted/practiced untouchability in varied forms. Hence the correct answer is (d). Source: Vajiram and Ravi Yellow book- “Modern Indian History”- Chapter 11, page no. 272. 35. The 19th and 20th century saw the growth of socio-religious movement among the

Dalits. Match the following movements with the region of activity:

List-I (Movement)

List-II (Region/State)

A Adi Dharma Movement 1 Chhattisgarh

B Satnam Panth 2 Kerala

C Balahari Movement 3 Punjab

D Sri Narayan Dharma Paripalana Yogam 4 Bengal

Codes A B C D (a) 3 1 4 2 (b) 1 2 3 4 (c) 4 3 2 1 (d) 3 2 1 4

Answer: (a) Explanation: In the early 20th century, there came into existence a number of socio reform movements among Dalits.

https://telegram.me/UPSCMaterials https://telegram.me/FreeUPSCMaterials https://telegram.me/MaterialforExam

Page 26: Answer Key & Exp for GS Prelim Test-5 held on 23rd …...1 Prelim IAS Test Series (2019) – GS Test 5 (23.12.2018) History 1 and Current Affairs (July 2018) Answer Key & Exp for GS

26

Prelim IAS Test Series (2019) – GS Test 5 (23.12.2018) History 1 and Current Affairs (July 2018)

Sri Narayan Dharma Paripalana Yogam was founded in 1903 and is a charitable society working for the spiritual and educational upliftment of the Ezhava community. Yogam is the first organisation which envisaged Kerala as a whole.

Adi Dharam Movement among the Chamars of Punjab

Adi Movement in Punjab.

Satnam Panth in Chhattisgarh

Balahari Movement in Bengal

Depressed Class Movement by Ambedkar Hence, the correct alternative is (a). Source: Vajiram and Ravi Yellow book- “Modern Indian History”- Chapter 11, page no. 271. 36. Consider the following events with respect to the evolution of two nation theory

during the freedom struggle: (1) Separate electorates were awarded under Morley-Minto reforms. (2) The first session of All India Hindu Mahasabha was held. (3) Agha Khan led a Muslim delegation to the Viceroy, Lord Minto. (4) Muslims participated in Rowlatt and Khilafat agitations. Arrange the events given above chronologically from earliest to latest: (a) 1-3-2-4 (b) 1-2-4-3 (c) 3-1-2-4 (d) 4-1-3-2

Answer: (c) Explanation: Evolution of two-nation theory:

1906 – Agha Khan led a Muslim delegation (called the Shimla Delegation) to Lord Minto to demand separate electorates.

1907 – All India Muslim League was founded by Agha Khan, Nawab Salimullah of Dacca, Nawab Mohsin-ul-Mulk, Nawab Waqar-ul-Mulk.

1909 – Separate electorates were awarded under Morley-Minto reforms.

1915 – First session of All India Hindu Mahasabha was held.

1916 – Congress accepted the Muslim League demand of separate electorates.

1920-22 – Muslims participated in Rowlatt-Khilafat satyagraha.

1932 – Communal Award accepted all 14 communal demands in 14 points. Hence the correct choice is (c). Source: Vajiram and Ravi Yellow book- “Modern Indian History”- Chapter 11, page no. 260-261. 37. ‘Self-respect Movement’ represented a radical trend in the non-Brahmin

movements. Consider the following statements in this regard: (1) Sri Narayan Guru pioneered this movement in South India.

https://telegram.me/UPSCMaterials https://telegram.me/FreeUPSCMaterials https://telegram.me/MaterialforExam

Page 27: Answer Key & Exp for GS Prelim Test-5 held on 23rd …...1 Prelim IAS Test Series (2019) – GS Test 5 (23.12.2018) History 1 and Current Affairs (July 2018) Answer Key & Exp for GS

27

Prelim IAS Test Series (2019) – GS Test 5 (23.12.2018) History 1 and Current Affairs (July 2018)

(2) The movement mounted scathing attacks on the Sanskrit language as a symbol of Aryan colonization.

Which of the statements given above is/are correct? (a) 1 only (b) 2 only (c) Both 1 and 2 (d) Neither 1 nor 2

Answer: (b) Explanation: The Self-Respect Movement was a movement with the aim of achieving a society where backward castes have equal human rights, and encouraging backward castes to have self-respect in the context of a caste-based society that considered them to be at the lower end of the hierarchy. The more radical movement within the non-Brahmin movements emerged in South India as the Self Respect Movement. Statement 1 is incorrect: It was spearheaded by E.V.Ramaswamy Naicker, also called “Periyar." He said before self-rule what was needed was self-respect. The movement was extremely influential not just in Tamil Nadu, but also overseas in countries with large Tamil populations, such as Malaysia and Singapore. Among Singapore Indians, groups like the Tamil Reform Association, and leaders like Thamizhavel G. Sarangapani were prominent in promoting the principles of the Self-Respect Movement among the local Tamil population through schools and publications. Statement 2 is correct: It mounted scathing attacks on the Sanskrit language as the symbol of Aryan colonization of the south. It was argued by the proponents of self-respect marriage that the then conventional marriages were officiated by Brahmins, who had to be paid for and also the marriage ceremony was in Sanskrit which most people did not understand, and hence were rituals and practices based on blind adherence. The Self-Respect movement encouraged inter-caste marriages, replacing arranged marriages by love marriages that are not constrained by caste. Source: Vajiram and Ravi Yellow book- “Modern Indian History”- Chapter 11, page no. 270-271. 38. Indian National Congress, though at first inclined to welcome Lord Curzon as a

reforming Viceroy, ended up taking an unfavourable view of his activity in India. Which of the following is/are concurrent with Lord Curzon’s regime? (1) Indian Official Secrets Act was enacted. (2) The British sent an expedition to Tibet. (3) Swadeshi Movement was started. Select the correct answer using the code given below: (a) 1 and 2 only (b) 3 only (c) 1 and 3 only (d) 1, 2 and 3

https://telegram.me/UPSCMaterials https://telegram.me/FreeUPSCMaterials https://telegram.me/MaterialforExam

Page 28: Answer Key & Exp for GS Prelim Test-5 held on 23rd …...1 Prelim IAS Test Series (2019) – GS Test 5 (23.12.2018) History 1 and Current Affairs (July 2018) Answer Key & Exp for GS

28

Prelim IAS Test Series (2019) – GS Test 5 (23.12.2018) History 1 and Current Affairs (July 2018)

Answer: (d) Explanation: Statement 1 is correct. Lord Curzon served as Governor General/ Viceroy of India from 6 January 1899 to 18 November 1905. His regime was very eventful. During Lord Curzon’s times, the Indian Official Secrets Act (1904) was enacted mainly to restrict the freedom of the press. Statement 2 is correct. At the end of 1903, Curzon sent a British expedition to Tibet. It is known as the Younghusband expedition. The expedition was intended to counter Russia's perceived ambitions in the East. It ended with The Anglo-Tibetan Treaty of Lhasa (1904) Statement 3 is correct. Partition of Bengal led to a Swadeshi Movement. It was announced during the regime of Curzon. Source: Vajiram and Ravi Yellow book- “Modern Indian History”- Chapter 8, Page 148-149. 39. The Home Rule league was ‘self-government’ based on the Irish model. Tilak and

Besant started the league separately with a convergent goal. In this context, consider the following statements: (1) Annie Besant secured Congress commitment for the Home Rule League. (2) Tilak’s League was set up after Annie Besant’s League. (3) Tilak’s League had more branches as compared to Annie Besant’s League. Which of the statements given above is/are correct? (a) 1 only (b) 3 only (c) 2 only (d) None of the above

Answer: (d) Explanation: Statement 1 is incorrect. Annie Besant launched the campaign for self-government for India along the line of white colonies like Ireland. This campaign was first limited to her newspaper like Commonweal; New India. Besant tried to get Congress’ approval the League but failed in her attempt. Statement 2 is incorrect. Tilak had set-up the League in April 1916 while Besant’s League was set-up in September 1916 Statement 3 is incorrect. Tilak’s League was restricted to Maharashtra (except Bombay), Karnataka, Central Province and Berar. It had six branches. Besant’s League had 200 branches. Source: Vajiram and Ravi Yellow book- “Modern Indian History”- Chapter 8, Page 162. 40. With reference to the Delhi-Lahore Conspiracy case, consider the following

statements: (1) It was aimed at assassination of the Viceroy, Lord Hardinge. (2) Ras Behari Bose was the brain behind the conspiracy.

https://telegram.me/UPSCMaterials https://telegram.me/FreeUPSCMaterials https://telegram.me/MaterialforExam

Page 29: Answer Key & Exp for GS Prelim Test-5 held on 23rd …...1 Prelim IAS Test Series (2019) – GS Test 5 (23.12.2018) History 1 and Current Affairs (July 2018) Answer Key & Exp for GS

29

Prelim IAS Test Series (2019) – GS Test 5 (23.12.2018) History 1 and Current Affairs (July 2018)

Which of the statements given above is/are correct? (a) 1 only (b) 2 only (c) Both 1 and 2 (d) Neither 1 nor 2

Answer: (c) Explanation: Statement 1 is correct. The Conspiracy was hatched in 1912. There was an attempt to assassinate the Viceroy using handmade bomb while a ceremonial procession was in place in Delhi. Statement 2 is correct. Ras Behari Bose along with Sachin Sanyal was the key conspirator in the case. He was able to evade arrest and flee to Japan. Source: Vajiram and Ravi Yellow book- “Modern Indian History”- Chapter 8, Page 158.

41. The history of the Ghadr movement was a testimony of the deep love that the Indian

immigrants had for India. Regarding the Ghadr movement, consider the following statements: (1) It was founded during the World War I for the furtherance of revolutionary

activities. (2) It was headquartered at San Francisco. (3) Their members were from Sikhs and Hindus only. Which of the statements given above is/are correct? (a) 2 only (b) 2 and 3 only (c) 1 and 2 only (d) 1, 2 and 3

Answer: (a) Explanation: Statement 1 is incorrect. Ghadr Movement began, in 1913, as a coalition of expatriate Punjabis dedicated to raising money and support for the overthrow of British rule in India. It was organized and headed by a Punjabi Hindu, Lala Har Dayal. It was founded before the outbreak of World War 1 Statement 2 is correct. Its headquarter was located at San Francisco Statement 3 is incorrect. Ghadr Movement had membership across all communities. Apart from leaders like Sohan Singh Bhakna and Bhai Parmanad, it included Muslims leaders Maulavi Barkatullah, Rehmat Ali and so on. Source: Vajiram and Ravi Yellow book- “Modern Indian History”- Chapter 8, Page 158-159. 42. With reference to the National Liberal Federation of India, consider the following

statements:

https://telegram.me/UPSCMaterials https://telegram.me/FreeUPSCMaterials https://telegram.me/MaterialforExam

Page 30: Answer Key & Exp for GS Prelim Test-5 held on 23rd …...1 Prelim IAS Test Series (2019) – GS Test 5 (23.12.2018) History 1 and Current Affairs (July 2018) Answer Key & Exp for GS

30

Prelim IAS Test Series (2019) – GS Test 5 (23.12.2018) History 1 and Current Affairs (July 2018)

(1) Tej Bahadur Sapru was one of the leaders of the National Liberal Federation of India.

(2) It supported the Montford reforms. Which of the statements given above is/are correct? (a) 1 only (b) 2 only (c) Both 1 and 2 (d) Neither 1 nor 2

Answer: (c) Explanation: Statement 1 is correct. National Liberal Federation of India was founded in 1918. It was a split of the liberal section of the Congress which supported August declaration by Montague. S.N Banerjee led it. Tej Bahadur Sapru along with V.S.S Shastri and M.R Jayakar was one of the leaders of the National Liberal Federation of India. Statement 2 is correct. The nationalists criticized Montford reforms, as it lacked their legitimate expectations. The declaration was criticized in the December 1917 Calcutta session, in which Annie Besant as president – pleaded for the establishment of Self Government in India. However, National Liberal Federation of India welcomed the reforms as it provided for responsible government in the provinces. Source: Vajiram and Ravi Yellow book- “Modern Indian History”- Chapter 8, Page 165. 43. With respect to the Komagata Maru incident of 1914, consider the following

statements: (1) The Komagata Maru ship sailed from Hong Kong to Canada. (2) The ship was not allowed to disembark at Vancouver port. (3) Ghadrites worldwide led a campaign against the event. Which of the statements given above is/are correct? (a) 2 only (b) 2 and 3 only (c) 1 and 2 only (d) 1, 2 and 3

Answer: (d) Explanation: Statement 1 is correct. Komagata Maru sailed from British Hong Kong, via Shanghai, China, and Yokohama, Japan, to Vancouver, British Columbia, Canada, in 1914. Statement 2 is correct. The ship was not allowed disembarking at Canada, and the ship was forced to return to India. The ship had more than 350 passengers. Statement 3 is correct. In a number of meetings ranging from California in 1914 to the Indian diaspora, prominent Ghadarites including Barkatullah, Tarak Nath Das, and Sohan Singh used the incident as a rallying point to recruit members for the Ghadar movement, most notably in support of promulgating plans to coordinate a massive uprising in India

https://telegram.me/UPSCMaterials https://telegram.me/FreeUPSCMaterials https://telegram.me/MaterialforExam

Page 31: Answer Key & Exp for GS Prelim Test-5 held on 23rd …...1 Prelim IAS Test Series (2019) – GS Test 5 (23.12.2018) History 1 and Current Affairs (July 2018) Answer Key & Exp for GS

31

Prelim IAS Test Series (2019) – GS Test 5 (23.12.2018) History 1 and Current Affairs (July 2018)

Source: Vajiram and Ravi Yellow book- “Modern Indian History”- Chapter 8, Page 159 44. With reference tothe Berlin Committee, consider the following statements:

(1) Virendranath Chatterjee was a key member of the Committee. (2) It planned an armed expedition against British India led by revolutionaries

within India. Which of the statements given above is/are correct? (a) 1 only (b) 2 only (c) Both 1 and 2 (d) Neither 1 nor 2

Answer: (c) Explanation: Statement 1 is correct. The Berlin Committee was an organisation formed in Germany in 1914 during World War 1 by Indian students and political activists residing in the country. Virendranath Chatterjee was a key member of the Committee. Statement 2 is correct. The Committee conceived a plan of the expedition against the British largely supported by an armed rebellion by Indian revolutionaries. It was part of Zimmerman plan as well. Source: Vajiram and Ravi Yellow book- “Modern Indian History”- Chapter 8, Page 161.

45. With reference to the revolutionaries, which of the following pair(s) is/are correctly

matched?

Organisation Founder

1 Bharat Mata Association : Vanchi Aiyer

2 Anjuman-i-Mohisbhan-i-Watan : Ajit Singh

3 Swadesh Sevak Home : Guran Ditt Kumar

Select the correct answer using the code given below: (a) None of the above (b) 3 only (c) 2 and 3 only (d) 1, 2 and 3

Answer: (d) Explanation: Pair 1 is correctly matched. Bharat Mata Association was established in 1904. Vanchi Aiyer was among its founder. In 1911, the District Magistrate of Tinnevali, Asche was shot dead by him. Pair 2 is correctly matched. Anjuman-i-Mohisbhan-i-Watan was a secret society established by Ajit Singh in 1906.

https://telegram.me/UPSCMaterials https://telegram.me/FreeUPSCMaterials https://telegram.me/MaterialforExam

Page 32: Answer Key & Exp for GS Prelim Test-5 held on 23rd …...1 Prelim IAS Test Series (2019) – GS Test 5 (23.12.2018) History 1 and Current Affairs (July 2018) Answer Key & Exp for GS

32

Prelim IAS Test Series (2019) – GS Test 5 (23.12.2018) History 1 and Current Affairs (July 2018)

Pair 3 is correctly matched. G.D. Kumar set up a Swadesh Sevak Home in Vancouver on the lines of the India House in London and also began to bring out a Gurumukhi paper called Swadesh Sewak which advocated social reform and also asked Indian troops to rise in revolt against the British. Source: India's Struggle For Independence, Bipin Chandra, page 151. 46. Consider the following statements about a revivalist movement:

(1) This movement was essentially an Islamic revivalist movement. (2) Jihad was declared against the Sikh Kingdom (3) Dar-ul-Harb was to be converted into Dar-ul-Islam. Which of the following movement is being referred in the statements given above? (a) Faraizi Movement (b) Wahabi Movement (c) Moplah Rebellion (d) Pagal Panthi Movement

Answer: (b) Explanation: Wahabi Movement was an Islamic revivalist movement founded by Syed Ahmed of Rai Bareilly who was inspired by the teaching of Abdul Wahab. It tried to purify Islam by eliminating all the un Islamic practices which had crept in Muslim society through the ages. Syed Ahmed had a countrywide organisation with an elaborate secret code for its working. It was strong in Sithana in the North Western tribal belt and at Patna though it had its missions in Hyderabad, Madras, Bengal, Uttar Pradesh and Bombay. Since dar-ul-Harb was to be converted to dar-ul-Islam, jihad was declared against the Sikh kingdom of Punjab. Peshawar was captured in 1830, but lost to the Sikhs in the following year with Syed Ahmad losing his life in action in the Battle of Balakot against the Sikhs (1831). After his death, Patna became the centre of this movement. Source: Vajiram and Ravi Yellow book- “Modern Indian History”- Chapter 6, Page no. 106-107. 47. Not only the peasants, but the deposed rulers and zamindars also rose up against

the East India Company’s policies. In this context which of the following factors accentuated the rising up of old zamindars and rulers? (1) Aggressive policy of annexation (2) The burden on economic revenue Select the correct answer using the code given below: (a) 1 only (b) 2 only (c) Both 1 and 2 (d) Neither 1 nor 2

Answer: (c) Explanation: Both the factors given in the question are correct.

https://telegram.me/UPSCMaterials https://telegram.me/FreeUPSCMaterials https://telegram.me/MaterialforExam

Page 33: Answer Key & Exp for GS Prelim Test-5 held on 23rd …...1 Prelim IAS Test Series (2019) – GS Test 5 (23.12.2018) History 1 and Current Affairs (July 2018) Answer Key & Exp for GS

33

Prelim IAS Test Series (2019) – GS Test 5 (23.12.2018) History 1 and Current Affairs (July 2018)

Point 1 is correct: The aggressive policy of annexation effected movements by deposed rulers and zamindars by the British and the subsequent disturbance in the economy of the regions. Pont 2 is correct: The movement was also accentuated by exacting land revenues. These uprisings are mainly known either by the name of the leader or the region where they occurred. Velu Thampi and Polygar Rebellions are two main uprisings in this category. Incidentally, Nana Sahib, Tantia Tope, and Begum Hazrat Mahal were all deposed rulers. Kunwar Singh was an unsatisfied zamindar. Source: Vajiram and Ravi Yellow book- “Modern Indian History”- Chapter 6, Page no. 106-107. 48. Which of the following statements is correct about a revolt by Velu Thampi during

1808-09? (a) He rose up in rebellion against the British attempt to remove him from the

Dewanship of Mysore. (b) Subsidiary Alliance System was still not effective in the area of his Dewanship. (c) His rebellion was not as prolonged as he was injured in a skirmish and died

soon. (d) None of the above

Answer: (c) Explanation: Option (a) is incorrect. Velu Thampi, the dewan of Travancore rose up in rebellion against the British attempts because the British attempted to remove him from the Dewanship Options (b) is incorrect. There was a heavy burden imposed on the state through the Subsidiary Alliance System. Option (c) is correct. In one of the ensuing skirmishes he was injured, and he died. The rebellion was limited to only 1808-09 and not a prolonged one. Source: Vajiram and Ravi Yellow book- “Modern Indian History”- Chapter 6, Page no. 106-107. 49. The Polygars in Kurnool were a class which could be comparable to the Rajputs of

North India. Which of the following statements is/are correct about this class? (1) They were the offshoots of the Nayankara System prevalent in the Vijaynagar

Administration. (2) They were given land in exchange for military service provided by them (3) They were not concerned with the extracting of the taxes. Select the correct answer using the code given below: (a) 1 and 2 only (b) 2 only (c) 3 only (d) None of the above

Answer: (a)

https://telegram.me/UPSCMaterials https://telegram.me/FreeUPSCMaterials https://telegram.me/MaterialforExam

Page 34: Answer Key & Exp for GS Prelim Test-5 held on 23rd …...1 Prelim IAS Test Series (2019) – GS Test 5 (23.12.2018) History 1 and Current Affairs (July 2018) Answer Key & Exp for GS

34

Prelim IAS Test Series (2019) – GS Test 5 (23.12.2018) History 1 and Current Affairs (July 2018)

Explanation: Statement 1 is correct. Polygar was the feudal title for a class of territorial administrative and military governors appointed by the Nayaka rulers of South India (notably Vijayanagara Empire, Madurai Nayakas and the Kakatiya dynasty) during 16th-18th centuries. Polygars were the offshoots of the Nayankara System prevalent in the Vijaynagar administration Statement 2 is correct. Polygars were given land in exchange for military service provided by them. They served as regional military and civil administrators. In turn they were to retain ¼ of the revenue collected as tax, and submit the remaining to the king's treasury. The Polygars also at times founded villages, built dams, constructed tanks and built temples. Also the rulers taxed regions according to the cultivability and fertility of the land. Often several new rainwater tanks were erected in the Semi-Arid tracts of western and south Tamil Nadu. Statement 3 is incorrect. Their influence and power increased beyond traditional lines, and they often acted as sovereigns even to the extent of extracting taxes from the people. The Company’s Government wanted to augment its sources of revenues, and it sought to control the Polygars. The conflict which aroused between Polygars and Company was on the question of the collection of taxes, rather, who should collect it, the traditional classes or the Company officials. Source: Vajiram and Ravi Yellow book- “Modern Indian History”- Chapter 6, Page no. 106-107. 50. Which of the following revolts/movements is also known as the “South Indian

Rebellion”? (a) Second Polygar War (b) First Polygar War (c) Ramosi Uprising (d) Moplah Rebellion

Answer: (a) Explanation: The Second Polygar war of 1800-1801, given the magnitude of participation, is also known as the South Indian Rebellion. It was directed by a confederacy consisting of Marudu Pandian of Sivaganga, Gopala Nayak of Dindigul, Kerala Verma of Malabar and Krishnappa Nayak and Dhoondaji of Mysore. The rebellion broke out when a band of polygar armies bombed the British barracks in Coimbatore in 1800. It took the British forces a year to quell the combined forces of the polygars. The suppression was followed by the signing of the Carnatic Treaty on July 31, 1801, whereby the British assumed direct control over Tamilnadu. The polygar system, which had flourished for two and a half centuries, came to a violent end and the Company introduced the Zamindari settlement in its place. Source: Vajiram and Ravi Yellow book - “Modern Indian History”- Chapter 6, Page no. 106-107. 51. Which of the following factors were responsible for the tribal uprisings in India?

https://telegram.me/UPSCMaterials https://telegram.me/FreeUPSCMaterials https://telegram.me/MaterialforExam

Page 35: Answer Key & Exp for GS Prelim Test-5 held on 23rd …...1 Prelim IAS Test Series (2019) – GS Test 5 (23.12.2018) History 1 and Current Affairs (July 2018) Answer Key & Exp for GS

35

Prelim IAS Test Series (2019) – GS Test 5 (23.12.2018) History 1 and Current Affairs (July 2018)

(1) Ban on shifting agriculture in reserved forests. (2) The growth of Royal Indian Navy and Railways (3) Introduction of the notion of private property (4) Influx of non-tribals in the tribal areas. Select the correct answer using the code given below: (a) 1, 2 and 4 only (b) 2 and 4 only (c) 1, 3 and 4 only (d) 1, 2, 3 and 4

Answer: (d) Explanation: All these factors were responsible for tribal uprisings in India 1. Shifting agriculture was a widespread practice among the tribals in India and banning

these under reserved forests was one of the main reasons for resentment. 2. The growth of Royal Indian Navy and Railways created a huge demand for wood.

Consequently forests were under the stricter regulation and control of the government, and the traditional rights of tribals inhabiting these forests were compromised.

3. Introduction of the notion of private property led to the commodification of land, i.e. it could be sold, bought and mortgaged. This, in turn, led to the loss of lands by tribals.

4. The influx of non-tribals in tribal areas reduced tribes to the status of agricultural labours.

Source: Vajiram and Ravi Yellow book- “Modern Indian History”- Chapter 6, Page-109. 52. Consider the following statements regarding the different phases of tribal

movements in India: (1) The first phase of tribal movements was generally led by people from lower

strata of society, whose rights over land were abolished. (2) The second phase of tribal movements was led by the upper strata of tribal

society whose position was undermined by the expansion of British rule. (3) The third phase of tribal movements got linked with the wider national

movements. Which of the statements given above is/are not correct? (a) 1 only (b) 1 and 2 only (c) 2 and 3 only (d) 1, 2 and 3

Answer: (b) Explanation: As soon as the British took over Eastern India, tribal revolts broke out to challenge alien rule. In the early years of colonisation, no other community in India offered such heroic resistance to British rule or faced such tragic consequences as did the numerous adivasi communities of now Jharkhand, Chhattisgarh, Odisha and Bengal.

https://telegram.me/UPSCMaterials https://telegram.me/FreeUPSCMaterials https://telegram.me/MaterialforExam

Page 36: Answer Key & Exp for GS Prelim Test-5 held on 23rd …...1 Prelim IAS Test Series (2019) – GS Test 5 (23.12.2018) History 1 and Current Affairs (July 2018) Answer Key & Exp for GS

36

Prelim IAS Test Series (2019) – GS Test 5 (23.12.2018) History 1 and Current Affairs (July 2018)

The first phase coincided with the establishment of the British Empire (1795 to 1860), the second phase with intensive colonialism during which merchant capital penetrated into tribal economy (1861 to 1920) and the third phase with the participation in the nationalist movement and also launching of agrarian as well as some separatist movements (1921 to 1947). Statement 1 is incorrect. The first phase of tribal movement saw the emergence of uprising from the upper strata of tribal society, whose privileges were undermined by the expansion of the British empire. Statement 2 is incorrect. The second phase of tribal movement was led by people belonging to the lower strata of the society when the tribal economy and the relationship with forests were undermined. Statement 3 is correct. The tribal movement in its third phase developed a tendency to get linked with the wider national movement as well as agrarian movements. In this phase, leadership was provided by diverse section ranging from Gandhian Social workers to Alluri Sitarama Raju. Source: Vajiram and Ravi Yellow book- “Modern Indian History” - Chapter 6, page - 109, 110, 111. 53. Consider the following statements regarding Munda Uprising:

(1) Introduction of Khuntkatti system by the Britishers was one of the major factors behind this uprising.

(2) The Munda uprising was primarily directed against the indigenous landlords and moneylenders.

Which of the statements given above is/are correct? (a) 1 only (b) 2 only (c) Both 1 and 2 (d) Neither 1 nor 2

Answer: (b) Explanation: Statement 1 is incorrect. Khuntkatti system was an already existing system among the tribals of present day Jharkhand through which land was held jointly through tribal lineages. This system started getting eroded due to the British rule in India, introduction of moneylenders and new class of landlords in tribal lands. Statement 2 is correct. The Munda Uprising was primarily directed against the indigenous landlords and moneylenders who were oppressing the tribal people. The Movement turned against the British as a result of their support to landlords and moneylenders. Source: Vajiram and Ravi Yellow book- “Modern Indian History”- Chapter 6, page-110 54. Which among the following can be adjudged as the causes of the failure of the

Revolt of 1857?

https://telegram.me/UPSCMaterials https://telegram.me/FreeUPSCMaterials https://telegram.me/MaterialforExam

Page 37: Answer Key & Exp for GS Prelim Test-5 held on 23rd …...1 Prelim IAS Test Series (2019) – GS Test 5 (23.12.2018) History 1 and Current Affairs (July 2018) Answer Key & Exp for GS

37

Prelim IAS Test Series (2019) – GS Test 5 (23.12.2018) History 1 and Current Affairs (July 2018)

(1) The Nawabs, princes and aristocracy were not able to provide an organised leadership against the British due to loss of influence and power among the general populace.

(2) The British already had an electric telegraph line installed by then, hence, constant monitoring of rebel movements was possible for the British commanders.

(3) Western-educated Indians supported the rebels. Select the correct answer using the codes given below: (a) 1 only (b) 1 and 2 only (c) 3 only (d) 2 and 3 only

Answer: (b) Explanation: Although the Revolt of 1857 was massive in its participation as well as its impact, the British had largely suppressed the Revolt of 1857 in a short span by pouring arms, men and money on a gigantic scale.

Those who participated lacked a clear understanding of the revolt and an alternative to what they were against.

Big zamindars backed off once the lands were restored.

Moneylenders helped restore British rule.

Mutineers were poorly equipped.

Eastern, southern and western parts remained largely unaffected. Statement 1 is correct: The Nawabs, princes and aristocracy were not able to provide an organised leadership although against the British due to loss of influence and power over the people. Statement 2 is correct: The British already had an electric telegraph line installed by then hence constantly monitoring the rebel movements was possible for the commanders. Statement 3 is incorrect: Western educated Indians supported the British rule believing that it will modernise the Indian society. Source: Vajiram and Ravi Yellow book- “Modern Indian History”- Chapter 7, page-110 55. The Revolt of 1857 marks the first major revolt with widespread participation. In this

context, which of the following statement is not correct? (a) Rumours of pig and cow fat in Enfield rifles were the immediate reasons for

triggering the revolt. (b) There was complete cooperation among the Hindus and Muslims at all levels. (c) The revolt broke out in the northern and central part of India. (d) The leaders of the Revolt had the vision of a unified India.

Answer: (d) Explanation:

https://telegram.me/UPSCMaterials https://telegram.me/FreeUPSCMaterials https://telegram.me/MaterialforExam

Page 38: Answer Key & Exp for GS Prelim Test-5 held on 23rd …...1 Prelim IAS Test Series (2019) – GS Test 5 (23.12.2018) History 1 and Current Affairs (July 2018) Answer Key & Exp for GS

38

Prelim IAS Test Series (2019) – GS Test 5 (23.12.2018) History 1 and Current Affairs (July 2018)

The first expression of organised resistance to the colonial rule was the Revolt of 1857. It began as a revolt of the sepoys of the Company’s army but eventually secured the participation of the masses.

The Revolt of 1857 eventually broke out over the incident of greased cartridges. A rumour spread that the cartridges of the new Enfield rifles were greased with the fat of cows and pigs.

Complete communal harmony was observed during the revolt.

Revolts broke out in northern and central India, and the outreach primarily remained in these areas itself. Eastern, Southern and western parts remained unaffected.

One of the reasons of failure was lack of a vision of unified India. There was no coherent ideology and an alternative blueprint of society.

Hence option (b) is the correct answer. Source: Vajiram and Ravi Yellow book- “Modern Indian History”- Chapter 7, Page 121-126. 56. Harappan seals provide a lot of insights into the past. They are used to reconstruct

which aspects of Harappan civilisation today? (1) Social dimensions (2) Political dimensions (3) Religious dimensions (4) Technological dimensions Select the correct answer using the code given below: (a) 1, 2 and 3 only (b) 1, 3, and 4 only (c) 2, 3 and 4 only (d) 1, 2, 3 and 4

Answer: (d) Explanation: Harappan seals are used to reconstruct all the given aspects of Harappan civilisation today. Statement 1 is correct. Social dimension of Harappan era can be gleaned from the various aspects on the seals. Their location, use and frequency of occurrence give some idea about their stratificaiton in the Harappan society – in the way that the society had unequal access to resources. Statement 2 is correct. It is said that the symbols on the seal not only represent the identity of individual merchants (if the seals are used for economic purposes) but also the insignia of various clans. These clans were not only social groupings but also possibly the political communities. Statement 3 is correct. The representation of various (real or mythical) animals, rituals, human-like figures and cultic symbols provide the glimpse of Harappa religion e.g. Pashupati seal, swastika symbol, peepal tree. Statement 4 is correct. Harappan seals give idea about the technological proficiency of Harappan people in two respects – in the way their material is prepared (steatite, carneial,

https://telegram.me/UPSCMaterials https://telegram.me/FreeUPSCMaterials https://telegram.me/MaterialforExam

Page 39: Answer Key & Exp for GS Prelim Test-5 held on 23rd …...1 Prelim IAS Test Series (2019) – GS Test 5 (23.12.2018) History 1 and Current Affairs (July 2018) Answer Key & Exp for GS

39

Prelim IAS Test Series (2019) – GS Test 5 (23.12.2018) History 1 and Current Affairs (July 2018)

jade, agate etc) and secondly, the way the artists and craftsmen work upon them. E.g. Ernstite used for drilling into hard material, micro-drilling etc. 57. The word ‘Brahman’ in the context of ancient India means which of the following?

(1) The superior Varna category (2) Part of books in the Vedic literature corpus (3) World soul or cosmic soul, the ultimate reality Select the correct answer using the code given below: (a) 1 and 2 only (b) 1 and 3 only (c) 2 and 3 only (d) 1, 2 and 3

Answer: (d) Explanation: Statement 1 is correct. Brahman is one of the four Varna categories – Brahmin, Kshatriya, Vaishya, Shudra. Statement 2 is correct. Brahman also represents a part of the Vedic corpus – they are a collection of ancient Indian texts with commentaries on the hymns of the four Vedas. They are a layer or category of Vedic Sanskrit texts embedded within each Veda and form a part of the Hindu Shruti literature. They are primarily a digest incorporating myths, legends, and the explanation of Vedic rituals and in some cases speculations about natural phenomenon or philosophy. The Brahmanas are particularly noted for their instructions on the proper performance of rituals, as well as explain the original symbolic meanings. Each Vedic shakha (school) has its own Brahmana. Numerous Brahmana texts existed in ancient India, many of which have been lost. A total of 19 Brahmanas are still existing at least in their entirety. Statement 3 is correct. Brahman also connotes the highest Universal Principle, the Ultimate Reality in the universe. In major schools of Hindu philosophy, it is the material, efficient, formal and final cause of all that exists. It is the pervasive, genderless, infinite, eternal truth and bliss which does not change, yet is the cause of all changes. Brahman as a metaphysical concept is the single binding unity behind diversity in all that exists in the universe. Brahman is a key concept found in the Vedas, and it is extensively discussed in the early Upanishads. The Vedas conceptualize Brahman as the Cosmic Principle. In the Upanishads, it has been variously described as Sat-chit-Ananda (truth-consciousness-bliss) and as the unchanging, permanent, highest reality. 58. Yajna/Sacrifice has been a Vedic tradition which evolved from simple offering

oblations into sacred fire to symbolic offerings in the presence of sacred Agni. They ranged from simple domestic rituals to majestic and royal kingship sacrifices. In this context, which of the following pair(s) is/are correctly matched?

Yajna Purpose

1 Vajpeya : To prove royal imperial sovereignty

https://telegram.me/UPSCMaterials https://telegram.me/FreeUPSCMaterials https://telegram.me/MaterialforExam

Page 40: Answer Key & Exp for GS Prelim Test-5 held on 23rd …...1 Prelim IAS Test Series (2019) – GS Test 5 (23.12.2018) History 1 and Current Affairs (July 2018) Answer Key & Exp for GS

40

Prelim IAS Test Series (2019) – GS Test 5 (23.12.2018) History 1 and Current Affairs (July 2018)

2 Ashvamedha : Associated with the attainment of power and prosperity

3 Rajasuya : Royal consecration ceremony

Select the correct answer using the code given below: (a) 1 only (b) 1 and 2 only (c) 3 only (d) 1, 2 and 3

Answer: (c) Explanation: Pair 1 is incorrect. The Vajapeya Sacrifice was connected with the attainment of power and prosperity, and also contained a number of fertility rites. It included a ritual chariot race in which the king used to race against his kinsmen and defeated them. Pair 2 is incorrect. The Ashvamedha was a sacrifice associated with claims to political paramountcy and incorporated several fertility rites as well. It was used by ancient Indian kings to prove their imperial sovereignty: a horse accompanied by the king's warriors would be released to wander for a period of one year. In the territory traversed by the horse, any rival could dispute the king's authority by challenging the warriors accompanying it. After one year, if no enemy had managed to kill or capture the horse, the animal would be guided back to the king's capital. It would be then sacrificed, and the king would be declared as an undisputed sovereign. Ashvameda is a forbidden rite for Kaliyuga, the current age. Pair 3 is correct. The Rajasuya was the royal consecration ceremony. Apart from a number of agrarian fertility rites, it included a ritual cattle raid, in which the rajan raided the cattle of his kinsmen. It also involves a game of dice, which the king wins and thus he is enthroned. At a larger, symbolic level, in the rajasuya, the king was presented as standing in the centre of the cyclical processes of regeneration of the universe. 59. Muktika Canon, an important book regarding the ancient Indian Hindu religious

texts, is a compilation of which of the following books? (a) Upanishads (b) Puranas (c) Various Vedantic Schools (d) Bhagvat Kathas of various regions

Answer: (a) Explanation: Muktika canon refers to the canon of 108 Upaniṣhads. The date of composition of each is unknown, with the oldest probably from about 800 BCE and the youngest probably composed during Mughal era. The canon is part of a dialogue between Rama and Hanuman where Rama proposes to teach Vedanta to Hanuman. 13 Upanishads are listed here as mukhya/principal Upanishads. Some of the Upanishads are categorized as

https://telegram.me/UPSCMaterials https://telegram.me/FreeUPSCMaterials https://telegram.me/MaterialforExam

Page 41: Answer Key & Exp for GS Prelim Test-5 held on 23rd …...1 Prelim IAS Test Series (2019) – GS Test 5 (23.12.2018) History 1 and Current Affairs (July 2018) Answer Key & Exp for GS

41

Prelim IAS Test Series (2019) – GS Test 5 (23.12.2018) History 1 and Current Affairs (July 2018)

"sectarian" since they present their ideas through a particular god or goddess such as Vishnu, Shiva, Shakti, or their combination. Thirteen Mukhya Upanishads – Isha, Kena, Katha, Prashna, Muṇḍaka, Mandukya, Taittiriya, Aitareya, Chandogya, Bṛhadaraṇyaka, Shvetashvatara, Kaushitaki, Maitri. There is no other such standardised compilation and canonization of Puranas or Vedantic schools or Bhagvat Kathas. 60. Rajatarangini, a historical chronicle of early India, was written in Sanskrit. It is

considered to be the best and most authentic work of its kind. It covers the entire span of history in the Kashmir region from the earliest times to the date of its composition. However, it is not alone. There are other books in a similar vein that give the glimpse of the history of Kashmir. Which of the books given below fall into the category of Rajatarangini? (1) Rajatarangini (by Jonaraja) (2) Jainatarangini (by Shrivara) (3) Rajavalipataka (by Prajyabhatta and Shuka) Select the correct answer using the code given below: (a) 1 and 2 only (b) 1 and 3 only (c) 2 and 3 only (d) 1, 2 and 3

Answer: (d) Explanation: The history of Kashmir was continued, along Kalhana’s line, down to some years after the annexation of Kashmir by the Mughal emperor Akbar (1586) in the following works: Rajatarangini (by Jonaraja), Jainatarangini (by Shrivara), and Rajavalipataka (by Prajyabhatta and Shuka). Neither in style nor authenticity do these works approximate the quality of Kalhana’s Rajatarangini. Statement 1 is correct. During the reign of Zain-ul-Abidin, Jonaraja authored a sequel by the same name. Also known as Dvitiya Rajatarangini, it gives an account of Kashmir from c. 1150 CE to 1459 CE. Statement 2 is correct. After Jonaraja's death in 1459, his disciple Srivara Pandita continued his work. He titled his work Jaina-Rajatarangini, and it is also known as Tritiya Rajatarangini ("third Rajatarangini"). It gives an account of Kashmir from 1459 CE to 1486 CE. Statement 3 is correct. Prajyabhatta's Rajavalipataka gives an account of Kashmir from 1486 to 1512. Suka extended Prajyabhatta's work, resulting in the Chaturtha Rajatarangini ("fourth Rajatarangini"). Suka's book ends with the arrival of Asaf Khan to Kashmir. A later interpolation also covers the arrival of the Mughal emperor Akbar and subsequent events. 61. Arrange the following pillars, which are historically significant for their inscriptions

and/or sculpture, in the chronological order of formation/occurrence/sculpting: (1) Heliodorus’ Besnagar Pillar

https://telegram.me/UPSCMaterials https://telegram.me/FreeUPSCMaterials https://telegram.me/MaterialforExam

Page 42: Answer Key & Exp for GS Prelim Test-5 held on 23rd …...1 Prelim IAS Test Series (2019) – GS Test 5 (23.12.2018) History 1 and Current Affairs (July 2018) Answer Key & Exp for GS

42

Prelim IAS Test Series (2019) – GS Test 5 (23.12.2018) History 1 and Current Affairs (July 2018)

(2) Allahabad Pillar (3) Vijaya Stambha, Chittor Select the correct answer using the code given below: (a) 1, 2, 3 (b) 2, 1, 3 (c) 2, 3, 1 (d) 3, 1, 2

Answer: (a) Explanation: Heliodorus pillar is a stone column that was erected around 113 BCE in Vidisha. Heliodorus, a Greek ambassador of the Indo-Greek king Antialcidas to the court of the Shunga king Bhagabhadra. Historically, it is one of the earliest known inscription related to Vaishnavism in India. The pillar was surmounted by a sculpture of Garuda and was dedicated by Heliodorus to the god Vasudeva in front of the temple of Vasudeva. The locals today call it Khamba Baba. The Heliodorus pillar, being dated rather precisely to the period of the reign of Antialcidas (approximately 115-80 BCE), is an essential marker of the evolution of Indian art during the Sunga period. It is, following the Pillars of Ashoka, the next pillar to be associated clearly with a datable inscription. The motifs on the pillar are key in dating some of the architectural elements of the nearby Buddhist complex of Sanchi. The Allahabad pillar is one of the pillars of Ashoka, an emperor of the Maurya dynasty who reigned in the 3rd century BCE. While it is one of the few extant pillars that carry his edicts, it is particularly notable for containing later inscriptions attributed to the Gupta emperor, Samudragupta (4th century CE). Also engraved on the stone are inscriptions by the Mughal emperor, Jahangir, from the 17th century. The inscription attributed to the 4th century CE Gupta emperor Samudragupta follows immediately below the edicts of Ashoka. It is considered "the most important historical document of the classical Gupta age". It is in excellent Sanskrit, written in the more refined Gupta script (a later version of Brahmi) by the poet and minister, Harishena. The inscription is a panegyric praising Samudragupta and lists the political and military achievements of his reign including his expeditions to the south. It provides a unique snapshot of the Gupta Empire and its neighbours and is the source of much of what is known of the geopolitical landscape of that era. The Mughal-era inscription records an earlier visit in c. 1575 of Akbar's courtier, Birbal, on a pilgrimage to the Sangam, the confluence of the Ganga, Yamuna, and Sarasvati rivers. The Vijaya Stambha is an imposing victory monument located within Chittorgarh fort in Chittorgarh, Rajasthan, India. The tower was constructed by the Mewar king, Rana Kumbha, in 1448 to commemorate his victory over the combined armies of Malwa and Gujarat led by Mahmud Khilji. The tower is dedicated to Vishnu. 62. Silappadikaram is one of Five Great Tamil Epics. As a literary work, Silappadikaram

is held in high regard by the Tamil people. Consider the following statements about it: (1) The tradition holds that ascetic-prince Ilango Adigal is the author

https://telegram.me/UPSCMaterials https://telegram.me/FreeUPSCMaterials https://telegram.me/MaterialforExam

Page 43: Answer Key & Exp for GS Prelim Test-5 held on 23rd …...1 Prelim IAS Test Series (2019) – GS Test 5 (23.12.2018) History 1 and Current Affairs (July 2018) Answer Key & Exp for GS

43

Prelim IAS Test Series (2019) – GS Test 5 (23.12.2018) History 1 and Current Affairs (July 2018)

(2) The epic revolves around the figure of Kannagi (also called Kannaki) (3) The story involves the three Tamil kingdoms of the ancient era, which were

ruled by the Chola, Pandyan and Chera dynasties Which of the statements given above is not correct? (a) 1 only (b) 2 only (c) 3 only (d) None of the above

Answer: (d) Explanation: Statement 1 is correct. Silappadikaram (The Jewelled Anklet) is the earliest epic poem in Tamil, written in the 5th–6th century AD by Prince Ilanko Adikal (Ilango Adigal). He identifies himself as a Chera prince from the 2nd century CE. He was the younger brother of Senguttuvan, the reputed warrior-king. Statement 2 is correct. The Silappadikaram tells of the young merchant Kovalan’s marriage to the virtuous Kannaki (Kannagi), his love for the courtesan Matavi, and his consequent ruin and exile in Madurai, where he is unjustly executed after trying to sell his wife’s anklet to a wicked goldsmith who had stolen the queen’s anklet and charged Kovalan with the theft. The widow Kannaki comes to Madurai, proves Kovalan’s innocence, then tears off one breast and throws it at the kingdom of Madurai, which goes up in flames. Such is the power of a faithful wife - the third book deals with a king’s expedition to bring Himalayan stone for an image of Kannaki, now a goddess of chastity. Statement 3 is correct. The Silappadikaram is a fine synthesis of mood poetry in an ancient Tamil Śaṅgam tradition and the rhetoric of Sanskrit poetry, including the dialogues of Kalittokai (poems of unrequited or mismatched love), chorus folk song, descriptions of city and village, lovingly technical accounts of dance and music, and strikingly dramatic scenes of love and tragic death. One of the great achievements of Tamil genius, the Silappadikaram is a detailed poetic witness to Tamil culture, its varied religions, its town plans and city types, the commingling of Greek, Arab, and Tamil peoples, and the arts of dance and music. The story involves the three Tamil kingdoms of the ancient era, which were ruled by the Chola, Pandyan and Chera dynasties. 63. Which of the following group of scripts/languages appear on Ashoka's inscriptions?

(a) Prakrit, Aramaic and Greek (b) Prakrit, Kharoshthi and Aramaic (c) Kharoshthi, Prakrit, Aramaic and Greek (d) Sanskrit, Prakrit, Kharoshthi, Greek, Aramaic and Brahmi

Answer: (a) Explanation: Three languages were used in Ashokan inscription – Prakrit, Greek and Aramaic. Four scripts were used. Prakrit inscriptions were written in Brahmi and Kharoshthi scripts, which even a commoner could read and understand. The inscriptions found in the area of Pakistan are in the Kharoshthi script. Other Edicts are written in Greek or Aramaic. The

https://telegram.me/UPSCMaterials https://telegram.me/FreeUPSCMaterials https://telegram.me/MaterialforExam

Page 44: Answer Key & Exp for GS Prelim Test-5 held on 23rd …...1 Prelim IAS Test Series (2019) – GS Test 5 (23.12.2018) History 1 and Current Affairs (July 2018) Answer Key & Exp for GS

44

Prelim IAS Test Series (2019) – GS Test 5 (23.12.2018) History 1 and Current Affairs (July 2018)

Kandahar Rock Inscription is bilingual Greek-Aramaic. The Kandahar Greek Edict of Ashoka is in Greek only. Ashoka's edicts were the first written inscriptions in India after the ancient city of Harappa fell to ruin. 64. Jaidev was the author of the famous Sanskrit poem Geet Govind. In the context of

Geet Govind consider the following statements: (1) Geet Govind portrays that Radha is more important than Krishna himself (2) Ashta Nayika, the significant portrayal of eight moods, find its mention in it. (3) Some of Jaidev’s hymns are included in Guru Granth Sahib (4) Geet Govind is regularly used in the performance of Odissi dance today Which of the statements given above are correct? (a) 1, 2 and 3 only (b) 1, 2 and 4 only (c) 2, 3 and 4 only (d) 1, 2, 3 and 4

Answer: (d) Explanation: Jaidev, was a Sanskrit poet during the 12th century. He is most known for his epic poem Gita Govinda which concentrates on Krishna's love with Radha. This poem, which presents the view that Radha is greater than Krishna, is considered an important text in the Bhakti movement of Hinduism. The text elaborates the eight moods of Heroine, the Ashta Nayika, which has been an inspiration for many compositions and choreographic works in Indian classical dances. Its verses are ashtapadis, which describe the beauty of Lord Krishna and the love between Krishna and the gopis, are considered a masterpiece in esoteric spirituality and the theme of 'Divine romance'. Jaidev is the earliest dated author of hymns that are included in the Guru Granth Sahib. There are records narrating how Jayadeva's work had a profound influence on Guru Nanak during his visit to Puri. He also institutionalised the Devadasi system in Odia temples. As a result of the great poet's works, Odia temples began to incorporate a separate Natamandira, or dance hall, within their precincts for Mahari dance (an earlier version of Odissi) performances. 65. Samannaphala Sutta of the Buddhist Pali canon mentions shramanic traditions in

ancient India, their respective philosophies and their founders. Which of the following pairs are correctly matched?

1 Purana Kassapa : Niyativada

2 Ajita Keshakambalin

: Lokayata

3 Pakudha Katyayana

: Shaswatavada

4 Sanjay Belatthaputta

: Ajnana

https://telegram.me/UPSCMaterials https://telegram.me/FreeUPSCMaterials https://telegram.me/MaterialforExam

Page 45: Answer Key & Exp for GS Prelim Test-5 held on 23rd …...1 Prelim IAS Test Series (2019) – GS Test 5 (23.12.2018) History 1 and Current Affairs (July 2018) Answer Key & Exp for GS

45

Prelim IAS Test Series (2019) – GS Test 5 (23.12.2018) History 1 and Current Affairs (July 2018)

Select the correct answer using the code given below: (a) 1, 2 and 3 only (b) 1, 3 and 4 only (c) 2, 3 and 4 only (d) 1, 2, 3 and 4

Answer: (c) Explanation: The views of six samaṇa in the Pali Canon (based on the Buddhist text Samannaphala Sutta)

Shramana (samana)

View

Puraṇa Kassapa Amoralism: denies any reward or punishment for either good or bad deeds.

Makkhali Goshala (Ajivika)

Niyativada (Fatalism): we are powerless; suffering is pre-destined.

Ajita Kesakambali (Lokayata)

Materialism: live happily; with death, all is annihilated.

Pakudha Katyayana

Shashwatavada (Eternalism): Matter, pleasure, pain and the soul are eternal and do not interact.

Nigaṇṭha Nataputta (Jainism)

Restraint: be endowed with, cleansed by and suffused with the avoidance of all evil.

Sanjaya Belaṭṭhiputta (Ajnana)

Agnosticism: "I don't think so. I don't think in that way or otherwise. I don't think not or not not."

https://telegram.me/UPSCMaterials https://telegram.me/FreeUPSCMaterials https://telegram.me/MaterialforExam

Page 46: Answer Key & Exp for GS Prelim Test-5 held on 23rd …...1 Prelim IAS Test Series (2019) – GS Test 5 (23.12.2018) History 1 and Current Affairs (July 2018) Answer Key & Exp for GS

46

Prelim IAS Test Series (2019) – GS Test 5 (23.12.2018) History 1 and Current Affairs (July 2018)

(Suspension of judgement.)

66. The book of Bhagavad Gita is significant not only for its religio-philosophical merit

but also with respect to its centrality to the debates surrounding India’s freedom struggle. Consider the following statements in this context: (1) Lokmanya Tilak wrote Gitarahasya in which he expounded Gita as a book

preaching karmayoga. (2) Sri Aurobindo claimed that non-violence could not be an absolute value. What

struck Aurobindo in his study of Gita was its bold “Gospel of action.” (3) Gita for Gandhiji was not a historical work, but under the guise of physical

warfare, it described the duel that perpetually went on in the hearts of mankind and Gandhiji called it Anasakti Yoga.

Which of the statements given above are correct? (a) 1 and 2 only (b) 1 and 3 only (c) 2 and 3 only (d) 1, 2 and 3

Answer: (d) Explanation: Three great nationalists interpreted the karmayoga of Gita in their ways and arrived at three different interpretations and three different courses of action. Statement 1 is correct. Tilak was convinced that the Gita was essentially a text that preached karmayoga. The commentaries on it that went before him seemed to emphasize spiritual knowledge, devotion and renunciation and this became so dominant that the yoga of action appeared to recede into the background. Most commentaries on Gita, he found, highlighted renunciation as the primary teaching and action or karma yoga as secondary. He could not understand why so much stress had been laid on the attainment of moksha through Jnana or Bhakti when the whole aim of Krishna’s teachings to the dejected Arjuna was to motivate him to fight. Tilak felt that the great commentators on the Gita who had gone before him had a doctrine or hypothesis into the framework of which they tried to interpret the Gita. He was thinking chiefly of Sankaracharya, Ramanujacharya and Madhvacharya. Each he felt was trying to interpret the message of Gita from a preconceived theory but he wanted the doctrinal method to be given up. Arjuna’s dilemma is the supreme ethical dilemma. The Mahabharata within which the Gita is placed is a mine of ethical dilemmas of the kind experienced by Arjuna. Tilak points out, the very first advice of Sri Krishna to Arjun is that it is not proper to give up action on the ground that numerous difficulties arise in consideration of what should be done and what should not be done. For Tilak, the subject matter of the Gita is to show whether or not there are any means of ascertaining what course should be followed when a person is beset by ethical dilemmas and by what means to resolve them. Statement 2 is correct. For Sri Aurobindo also, non-violence could not be an absolute value. What struck Aurobindo in his study of Gita was its bold “Gospel of action” and its stress on the kshatriya’s “duty to protect the world from injustice." The virtue of the

https://telegram.me/UPSCMaterials https://telegram.me/FreeUPSCMaterials https://telegram.me/MaterialforExam

Page 47: Answer Key & Exp for GS Prelim Test-5 held on 23rd …...1 Prelim IAS Test Series (2019) – GS Test 5 (23.12.2018) History 1 and Current Affairs (July 2018) Answer Key & Exp for GS

47

Prelim IAS Test Series (2019) – GS Test 5 (23.12.2018) History 1 and Current Affairs (July 2018)

Brahmin is a great virtue: You shall not kill. This is what Ahimsa means. But if the virtue of Ahimsa comes to the Kshatriya, if you say, “I will not kill”, there is no one to protect the country. The happiness of the people will be broken down. Injustice and lawlessness will reign." The teaching of the Gita, he said in his concluding words, “means perfection of action. It makes a man great. It gives him the utter strength, the utter bliss which is the goal of life in the world.” Statement 3 is correct. Mahatma Gandhi took the path of non-violence and led the country to freedom through it. Gita for Gandhi was not a historical work, but under the guise of physical warfare, it described the duel that perpetually went on in the hearts of mankind. The physical warfare was only brought in to make the description of this internal duel more alluring. Renunciation did not mean that the renouncer did not get any fruits of his action. The Gita did not warrant such a meaning. Renunciation means the absence of hankering after fruit. The renunciation demanded by the Gita was the acid test of faith. He who was ever brooding over result often lost nerve in the performance of his duty. Thinking along these lines, Gandhi felt that if he had to enforce the central teaching of Gita in his own life, he was bound to follow Truth and Ahimsa. When there was no desire for fruit, there is no temptation for untruth or ahimsa. But he freely admitted that the Gita was not written to establish ahimsa. It was an accepted and primary duty even before the age of Gita. The Gita’s message was the renunciation of the fruit of action. Gandhi calls it Anasakti Yoga. 67. The Sanskrit language has a rich tradition of commentary on grammar (vyakarana).

Which of the following is the oldest known text on Sanskrit grammar? (a) Panini’s Ashtadhyayi (b) Patanjali’s Mahabhashya (c) Katyayana’s Varttika (d) Jayaditya’s Kashika

Answer: (a) Explanation: Paṇini was an ancient Sanskrit philologist, grammarian, and a revered scholar in ancient India. Considered the father of linguistics, Paṇini likely lived in the northwest Indian subcontinent during the Mahajanapada era. He is said to have been born in Shalatula of ancient Gandhara, which likely was near modern Lahore. He is known for his text Ashtadhyayi, a sutra-style treatise on Sanskrit grammar - rules on linguistics, syntax and semantics which is the foundational text of the Vyakaraṇa branch of the Vedanga. Nothing definite is known about when Paṇini lived, not even in which century he lived. Most scholarship places him in or before mid-fourth century BCE, possibly in the sixth or fifth century BCE. Katyayana (c. 300 BC) was a Sanskrit grammarian, mathematician and Vedic priest. His vartikas are an elaboration of Panini’s Ashtadhyayi. Along with the Mahabhaṣya of Patanjali, this text became a core part of the Vyakaraṇa (grammar) canon. Mahabhashya, attributed to Patanjali, is a commentary on selected rules of Sanskrit grammar from Paṇini's treatise, the Ashtadhyayi, as well as Katyayana's Varttika. It is dated to the 2nd century BCE.

https://telegram.me/UPSCMaterials https://telegram.me/FreeUPSCMaterials https://telegram.me/MaterialforExam

Page 48: Answer Key & Exp for GS Prelim Test-5 held on 23rd …...1 Prelim IAS Test Series (2019) – GS Test 5 (23.12.2018) History 1 and Current Affairs (July 2018) Answer Key & Exp for GS

48

Prelim IAS Test Series (2019) – GS Test 5 (23.12.2018) History 1 and Current Affairs (July 2018)

Kashika is a commentary on Paṇini, attributed to Jayaditya and Vamana, composed in the 7th century. It is considered the "fourth great grammar" of Sanskrit, after Panini himself (4th century BCE), Patanjali's Mahabhasya (2nd century BCE) and Bhartrhari's Vakyapadiya (6th century CE). The most influential work of the Early Modern period was Siddhanta-Kaumudi by Bhaṭṭoji Dikṣita. He was a 17th-century Maharashtrian Sanskrit grammarian. It has been described as "an encyclopedia of the opinions and views of the great Sanskrit grammarians of antiquity." He was active in a revival of the grammatical methods of Panini. 68. There are various traditions of philosophical schools in India. There are six major

astika schools, i.e. orthodox philosophy - Nyaya, Vaisheshika, Samkhya, Yoga, Mimamsa and Vedanta, and four major nastika, i.e. heterodox schools - Jain, Buddhist, Ajivika, and Charvaka. Which of the following criteria is/are used to make such astika-nastika classification of the various schools of philosophy in India? (1) Whether the school believes that the Vedas are a valid source of knowledge (2) Whether the school believes in the premises of Brahman and Atman Select the correct answer using the code given below: (a) 1 only (b) 2 only (c) Both 1 and 2 (d) Neither 1 nor 2

Answer: (c) Explanation: The principal schools are classified as either orthodox or heterodox - astika or nastika - depending on one of three alternate criteria: Whether it believes the Vedas are a valid source of knowledge, whether the school believes in the premises of Brahman and Atman, whether the school believes in afterlife and Devas. Different thinkers and philosophical divide the schools based on either or all of these three criteria. 69. Ajivika is one of the nastika or heterodox schools of Indian philosophy. Consider the

following statements regarding it: (1) The theories of Ajivika philosophy are extracted from secondary sources like

Buddhist and Jaina books, as the original scriptures are lost. (2) Ajivika school did not believe in the Niyati (Fate) doctrine of absolute

determinism (3) Ajivika metaphysics included a theory of atoms similar to the Vaisheshika (4) It reached the height of its popularity during the rule of the Mauryan emperor

Bindusara, around the 4th century BCE Which of the statements given above are correct? (a) 1, 2 and 3 only (b) 1, 3 and 4 only (c) 2, 3, and 4 only (d) 1, 2, 3 and 4

https://telegram.me/UPSCMaterials https://telegram.me/FreeUPSCMaterials https://telegram.me/MaterialforExam

Page 49: Answer Key & Exp for GS Prelim Test-5 held on 23rd …...1 Prelim IAS Test Series (2019) – GS Test 5 (23.12.2018) History 1 and Current Affairs (July 2018) Answer Key & Exp for GS

49

Prelim IAS Test Series (2019) – GS Test 5 (23.12.2018) History 1 and Current Affairs (July 2018)

Answer: (b) Explanation: Statement 1 is correct. Ajivika, an ascetic sect that emerged in India about the same time as Buddhism and Jainism and that lasted until the 14th century. It was founded by Goshala Maskariputra (also called Gosala Makkhaliputta), a friend of Mahavira, the 24th Tirthankara. His doctrines and those of his followers are known only from Buddhist and Jain sources, which state that he was lowborn and died after a quarrel with Mahavira shortly before the Buddha died. Statement 2 is incorrect. Ajivika School is known for its Niyati (Fate) doctrine of absolute determinism, the premise that there is no free will, that everything that has happened, is happening and will happen is entirely preordained and a function of cosmic principles. They considered the karma doctrine as a fallacy. Statement 3 is correct. Ajivika metaphysics included a theory of atoms similar to the Vaisheshika, where everything was composed of atoms, qualities emerged from aggregates of atoms, but the aggregation and nature of these atoms was predetermined by cosmic forces. Statement 4 is correct. Inscriptions in the Barabar hills record Ashoka’s dedication of some caves to Ajivika ascetics. In the nearby Nagarjuni hills, inscriptions record the dedication of three caves to them by Ashoka’s successor, Dasharatha. The Maurya period was the heyday of the Ajivika sect, but references to it continue in various sources till the early medieval period. 70. Bhakti, as it emerged in the classical sense, is of two types – Saguna and Nirguna.

Consider the following statements in this regard: (1) Saguna bhakti is looking at God with various attributes. (2) Nirguna bhakti is attributing God with universality. Which of the statements given above is/are not correct? (a) 1 only (b) 2 only (c) Both 1 and 2 (d) Neither 1 nor 2

Answer: (d) Explanation: The Bhakti movement of Hinduism saw two ways of imaging the nature of the divine (Brahman) - Nirguna and Saguna. Nirguna Brahman is the concept of the Ultimate Reality as formless, without attributes or quality. Saguna Brahman, in contrast, is envisioned and developed as with form, attributes and quality. It is the same Brahman but viewed from two perspectives, one from Nirguni knowledge-focus and other from Saguni love-focus. Nirguna bhakti is an unconditional devotion to God in an unmanifested form. Its opposite is saguna bhakti, which is worship of and devotion to a manifested deity who takes the form of an idol, an image or an incarnation. Saguna bhakti is easier to comprehend because the ishta devata, or personal god, is worshipped in a recognizable form. Nirguna bhakti, on the other hand, is devotion to the Absolute - the divine energy that is the universe and the highest consciousness.

https://telegram.me/UPSCMaterials https://telegram.me/FreeUPSCMaterials https://telegram.me/MaterialforExam

Page 50: Answer Key & Exp for GS Prelim Test-5 held on 23rd …...1 Prelim IAS Test Series (2019) – GS Test 5 (23.12.2018) History 1 and Current Affairs (July 2018) Answer Key & Exp for GS

50

Prelim IAS Test Series (2019) – GS Test 5 (23.12.2018) History 1 and Current Affairs (July 2018)

71. Alvars are considered the twelve supreme devotees of Vishnu, who were

instrumental in popularising Vaishnavism in the Tamil-speaking regions. Which among following is the only woman Alvar saint? (a) Andal (b) Nammalvar (c) Peyalvar (d) Thiruppan

Answer: (a) Explanation: Option (a) is correct. Andal is the only female Alvar among the 12 Alvar saints of South India. Andal is credited with the great Tamil works, Thiruppavai and Nachiar Tirumozhi, which are still recited by devotees during the winter festival season of Margazhi. Option (b) is incorrect. Nammalwar is considered the fifth in the line of the twelve alvars. He is highly regarded as a great mystic of the Vaishnava tradition. He is also considered the greatest among the twelve alvars. The works of Nammalwar contributed to the philosophical and theological ideas of Vaishnavism. Along with the three Shaiva Nayanars Appar, Sundarar and Sambandar, they influenced the ruling Pallava kings of South India, changing the religious geography from Buddhism and Jainism to Hinduism. Option (c) is incorrect. Peyalvar is considered third in the list of the three principal Alvars. He composed hundred verses that are classified as Moondram Tiruvandadhi Vishnu. The works of the earliest saints contributed to the philosophical and theological ideas of Vaishnavism. Option (d) is incorrect. Thiruppan is considered the eleventh in the line of the twelve alvars. As per Hindu legend, he descended from eternity and was found in a paddy field by a couple from the paanar community. Thiruppan is known for his affiliation to Ranganatha of the Srirangam Ranganathaswamy temple and is believed to have merged with the deity. In South-Indian Vishnu temples, Thiruppan has images and festivals associated with him. 72. The early medieval period saw the emergence and increasing popularity of the

Virashaiva or Lingayat movement. The great popularity it achieved in Karnataka was to a large extent the contribution of Basavanna. In this respect, identify the incorrect statement from the following: (a) Basava’s poetry is called as Vachans, which was instrumental in the spread of

Lingayatism. (b) Although Virashaivism doesn’t believe in Brahmin priestly class, they have a

system of monks called Jangam. (c) Basava didn’t believe in temple-based bhakti. (d) Anubhav Mantap is the Samadhi place of Basavanna.

Answer: (d) Explanation:

https://telegram.me/UPSCMaterials https://telegram.me/FreeUPSCMaterials https://telegram.me/MaterialforExam

Page 51: Answer Key & Exp for GS Prelim Test-5 held on 23rd …...1 Prelim IAS Test Series (2019) – GS Test 5 (23.12.2018) History 1 and Current Affairs (July 2018) Answer Key & Exp for GS

51

Prelim IAS Test Series (2019) – GS Test 5 (23.12.2018) History 1 and Current Affairs (July 2018)

Statement 1 is correct. Vachana Sahitya is a form of Kannada poetry that evolved in the 11th century CE and flourished in the 12th century. The Virashaivas accepted many of the doctrines of other Shaiva schools, but its core ideas are encapsulated in free verse lyrics known as vachanas, composed by the saints. The word vachanas means "(that which is) said". These are readily intelligible prose texts. Vachanas are brief paragraphs, and they end with one or the other local names under which Shiva is invoked or offered Pooja. The Vachanas call men to give up the desire for worldly wealth and ease, to live lives of sobriety and detachment from the world and to turn to Siva for refuge. Statement 2 is correct. The Jangams is a Shaiva order of wandering religious monks. They belong to the Virshaiva Lingayat community. They are the priests or gurus of the Hindu Shaiva. There is an age-old tradition calling Jangam as gurus of 'Lingayat’. The Jangam is the wandering holy man in Virashaivism. Statement 3 is correct. Basava championed devotional worship that rejected temple worship and rituals led by Brahmins and replaced it with personalised direct worship of Shiva. Worship is centred on Shiva as the universal god in the iconographic form of Ishtalinga They attach no importance to worshipping the god in temples. Statement 4 is incorrect. The Anubhava Mantapa literally means the "hall of spiritual experience". It has been a Lingayat institution since the time of Basava, serving as an academy of mystics, saints and poet-philosophers for discussion of spiritual and mundane questions of life, in the open. It was the fountainhead of all religious and philosophical thought about the Lingayata. It was presided over by the mystic Allamaprabhu, and numerous sharanas from all over Karnataka and other parts of India were participants. This institution also helped propagate Lingayatism religious and philosophical thought. Akka Mahadevi, Channabasavanna and Basavanna himself were participants in the Anubhava Mantapa. 73. Adi Shankaracharya, the prime mover behind Advaita Vedanta, composed many

commentaries and songs to spread his message far and wide. Which of the following are his compositions? (1) Vivekachudamani (2) Saundarya Lahari (3) Bhaja Govindam (4) Nirvana Shatakam Select the correct answer using the code given below: (a) 1, 2 and 3 only (b) 1, 3 and 4 only (c) 2, 3 and 4 only (d) 1, 2, 3 and 4

Answer: (d) Explanation: Vivekchudamani. It’s a Sanskrit verse. It has the form of dialogue where the master explains to the disciple the nature of the Atman and the ways to research and know the Atman. The book takes the disciple on a step by step instructions to reach Brahman. It talks about qualifications required by a good student to absorb the knowledge of Vedanta.

https://telegram.me/UPSCMaterials https://telegram.me/FreeUPSCMaterials https://telegram.me/MaterialforExam

Page 52: Answer Key & Exp for GS Prelim Test-5 held on 23rd …...1 Prelim IAS Test Series (2019) – GS Test 5 (23.12.2018) History 1 and Current Affairs (July 2018) Answer Key & Exp for GS

52

Prelim IAS Test Series (2019) – GS Test 5 (23.12.2018) History 1 and Current Affairs (July 2018)

Saundarya Lahari. It means the ‘waves of beauty’. Its hundred and three shlokas (verses) eulogize the beauty, grace and munificence of Goddess Parvati / Dakshayani, consort of Shiva. It is not only the collection of holy hymns, but also a tantra textbook. Bhaja Govindam. It’s a very popular devotional stotra by Shankara that underscores the view that Bhakti to God is an important part of general spirituality. It is generally considered a summary of Advaita Vedanta philosophy. In it, Shankara emphasizes the importance of devotion for God as a means to spiritual development and to liberation from the cycle of birth and death. It is a reminder that Shankara, who is often regarded as a stalwart advocate of the Jnana Marga also appreciated the Bhakti Marga. The rendition of this hymn by M.S. Subbulakshmi is very popular and appreciated for her immaculate pronunciation. Atma Shatakam (aka Nirvana Shatakam). It’s a composition by Shankara summarizing the basic teachings of Advaita Vedanta. It is said that when he was a young boy of eight and wandering near River Narmada, seeking to find his guru, he encountered the seer Govinda Bhagavatpada who asked him, "Who are you?". The boy answered with these stanzas. Every stanza enumerates what he is not and then ends with the phrase ‘chidanandarupahshivohamshivoham.’ Upon these, Swami accepted him as a disciple. 74. Consider the following statements regarding the bhakti saint-poets:

(1) Surdas, the great bhakti poet of his times, is considered as one of the Ashtachhap.

(2) Ashtachhap were 8 poets of medieval north India – 4 Shaiva and 4 Vaishnav – who were instrumental in spreading the idea of Bhakti.

Which of the statements given above is/are correct? (a) 1 only (b) 2 only (c) Both 1 and 2 (d) Neither 1 nor 2

Answer: (a) Explanation: In 1565, Vitthalnath (He was the son and successor of Vallabhacharya) established the eight-fold system of singing the name and glory of Shrinathji (Kirtana) and entrusted this responsibility to eight poet-disciples. These are Surdas, Krishna Das, Paramanand Das, Kumbhan Das, Chaturbhuj Das, Nand Das, Chhitswami, and Govind Das. The greatest of the group was Surdas, a blind singer whose descriptions of the exploits of the child-god Krishna are the highlights of his collection of poetry called the Sursagar, a work that is admired throughout the Hindi-speaking areas of northern India. It is particularly rich in its details of daily life and in its sensitive perception of human emotion. The Sur Sagar in its present form focuses on descriptions of Krishna as a lovable child, written from the gopis' perspective. However, many of the poems in the book seem to be written by later poets in Sur's name. Sur was also a great religious singer. Sur's poetry was written in a dialect of Hindi called Braj Bhasha, until then considered to be a very plebeian language, as the prevalent literary languages were either Persian or Sanskrit. His work raised the status of Braj Bhasha from a crude language to that of a literary one.

https://telegram.me/UPSCMaterials https://telegram.me/FreeUPSCMaterials https://telegram.me/MaterialforExam

Page 53: Answer Key & Exp for GS Prelim Test-5 held on 23rd …...1 Prelim IAS Test Series (2019) – GS Test 5 (23.12.2018) History 1 and Current Affairs (July 2018) Answer Key & Exp for GS

53

Prelim IAS Test Series (2019) – GS Test 5 (23.12.2018) History 1 and Current Affairs (July 2018)

75. Vari is an annual pilgrimage to Pandharpur - the seat of Vitthal. The tradition is more

than 700 to 800 years old. Arrange the following Varkari saint-poets in chronological order of their birth (earliest to latest). (1) Dnyaneshwar (2) Namdev (3) Eknath (4) Tukaram Select the correct order of the above poets: (a) 1 - 2 - 3 - 4 (b) 1 - 2 - 4 - 3 (c) 2 - 1 - 3 - 4 (d) 2 - 1 - 4 - 3

Answer: (c) Explanation: Dnyaneshwar (1275-1296) was the pioneer bhakti saint of Maharashtra. He was a 13th-century Marathi saint, poet, philosopher and yogi of the Nath tradition whose Dnyaneshwari (a commentary on the Bhagavad Gita) and Amrutanubhav are considered to be milestones in Marathi literature. This was one of the earliest works of Marathi literature and served as the foundation of bhakti ideology in Maharashtra. He was the author of many hymns called abhangs. Namdev (1270-1350) belonged to tailor caste. He is considered to be the link between the Maharashtrian bhakti movement and North Indian monotheistic movement. He lived in Pandharpur but travelled to North India including Punjab. He is also venerated in Sikhism (His 61 bhakti songs have been included in the Adi Granth) as well as Hindu warrior-ascetic traditions such as the Dadupanthis and the Niranjani Sampradaya that emerged in north India during the Islamic rule. His philosophy contains both nirguna and saguna Brahman elements, with monistic themes. In Maharashtra, Namdev is considered to be a part of the varkari tradition, but in the North Indian monotheistic tradition he is remembered as a nirguna saint. Namdev is considered one the five revered gurus in the Dadupanth tradition, the other four being Dadu, Kabir, Ravidas and Hardas. Eknath (1533-1599) was a prominent Marathi sant, scholar, and religious poet of the Varkari sampradaya. In the development of Marathi literature, Eknath is seen as a bridge between his predecessors - Dnyaneshwar and Namdev - and the later Tukaram and Ramdas. He wrote a variation of the Bhagavata Purana which is known as the Eknathi Bhagavata, and a variation of the Ramayana which is known as the Bhavarth Ramayan. Tukaram (1598-1650) was a 17th-century poet-saint of the Bhakti movement in Maharashtra. Tukaram is best known for his Abhanga devotional poetry and community-oriented worship with spiritual songs known as kirtans. He considered kirtan, not just a means to learn about Bhakti, but Bhakti itself. The greatest merit in kirtan, according to Tukaram, is it being not only a spiritual path for the devotee, but it also helps create a spiritual path for others. His poetry was devoted to Vitthala. 76. With reference to ‘Net Neutrality’, consider the following statements:

https://telegram.me/UPSCMaterials https://telegram.me/FreeUPSCMaterials https://telegram.me/MaterialforExam

Page 54: Answer Key & Exp for GS Prelim Test-5 held on 23rd …...1 Prelim IAS Test Series (2019) – GS Test 5 (23.12.2018) History 1 and Current Affairs (July 2018) Answer Key & Exp for GS

54

Prelim IAS Test Series (2019) – GS Test 5 (23.12.2018) History 1 and Current Affairs (July 2018)

(1) Every net user must have equal access to all lawful online content at the same speed and cost.

(2) There are no exceptions in the Net Neutrality principle. (3) India officially upholds the principles of net neutrality. Which of the statements given above is/are correct? (a) 2 only (b) 2 and 3 only (c) 1 and 3 only (d) 1 and 2 only

Answer: (c) Explanation: Statement 1 is correct: Net neutrality is the basic principle of an open Internet that does not allow for content discrimination by Internet Service Providers (ISPs). The user is free to access any web location at the same paid-for speed without any discrimination by the ISP. Statement 2 is incorrect: Services like autonomous driving, telemedicine or remote diagnostic services are an exception to net neutrality as they may require prioritised internet lane and faster than normal speed. Statement 1 is correct: Recently India officially upheld the principles of net neutrality. Telecom Commission has approved the recommendations of the Telecom Regulatory Authority of India (TRAI) on net neutrality. By endorsing steps that call for amendments to access services licences for Internet Service Providers (ISPs) and Telecom Operators, the Commission has made it clear that any violation of net neutrality will be treated as a violation of the licence conditions. Source: https://www.thehindu.com/opinion/editorial/a-welcome-move/article24427683.ece 77. Which of the following are key functions of the Chemical Weapons Convention

(CWC)? (1) Destroying all the existing chemical weapons under international verification

by the Organisation for the Prohibition of Chemical Weapons (OPCW). (2) Monitoring chemical industry to prevent new weapons from re-emerging. (3) Providing assistance and protection to States Parties against chemical

threats. (4) Fostering international cooperation to strengthen implementation of the

Convention and promote the peaceful use of chemistry. Select the correct answer using the code given below: (a) 2 only (b) 1 and 2 only (c) 1, 3 and 4 only (d) 1, 2, 3 and 4

Answer: (d) Explanation:

https://telegram.me/UPSCMaterials https://telegram.me/FreeUPSCMaterials https://telegram.me/MaterialforExam

Page 55: Answer Key & Exp for GS Prelim Test-5 held on 23rd …...1 Prelim IAS Test Series (2019) – GS Test 5 (23.12.2018) History 1 and Current Affairs (July 2018) Answer Key & Exp for GS

55

Prelim IAS Test Series (2019) – GS Test 5 (23.12.2018) History 1 and Current Affairs (July 2018)

All the statements are correct. The Convention aims to eliminate an entire category of weapons of mass destruction by prohibiting the development, production, acquisition, stockpiling, retention, transfer or use of chemical weapons by States Parties. States Parties, in turn, must take the steps necessary to enforce that prohibition in respect of persons (natural or legal) within their jurisdiction. The Organisation for the Prohibition of Chemical Weapons (OPCW) Member States share the collective goal of preventing chemistry from ever again being used for warfare, thereby strengthening international security. To this end, the Convention contains four key provisions: 1. Destroying all existing chemical weapons under international verification by the

OPCW. 2. Monitoring chemical industry to prevent new weapons from re-emerging. 3. Providing assistance and protection to States Parties against chemical threats. 4. Fostering international cooperation to strengthen implementation of the Convention

and promote the peaceful use of chemistry. Sources: https://www.thehindu.com/todays-paper/tp-opinion/the-man-with-the-clues/ article19255066.ece https://www.opcw.org/chemical-weapons-convention 78. Regarding ‘Project Sashakt’, which of the following statements is/are correct?

(1) It is to ensure the operational turnaround of the banks and stressed companies so that the asset value is retained.

(2) For assets up to `500 crores, an independent asset management company (AMC) would be created that will be funded by alternative investment funds (AIF).

Select the correct answer using the code given below: (a) 1 only (b) 2 only (c) Both 1 and 2 (d) Neither 1 nor 2

Answer: (c) Explanation: Both the statements are correct. A committee of representatives from several banks - led by the Punjab National Bank chairman Sunil Mehta - announced an alternative mechanism for resolving bad loans, called Project Sashakt. It is a bank led resolution approach (BLRA). According to report "Project Sashakt" is to ensure the operational turnaround of the banks and stressed companies so that the asset value is retained. Five-pronged resolution processes suggested by the committee will bring in credible long term capital to limit the burden on the domestic banking sector while preventing similar build-up of non-performing loans in the future using robust governance and credit architecture. Five-pronged resolution suggested by the committee:

https://telegram.me/UPSCMaterials https://telegram.me/FreeUPSCMaterials https://telegram.me/MaterialforExam

Page 56: Answer Key & Exp for GS Prelim Test-5 held on 23rd …...1 Prelim IAS Test Series (2019) – GS Test 5 (23.12.2018) History 1 and Current Affairs (July 2018) Answer Key & Exp for GS

56

Prelim IAS Test Series (2019) – GS Test 5 (23.12.2018) History 1 and Current Affairs (July 2018)

The committee has classified small, medium and large assets range from 0-50 Crores Rupee, 50-500 crore Rupee and 500 crore and more respectively, resolutions suggested by committee are applicable to all small, medium and large assets. These resolutions covering both performing and non-performing assets are also applicable to any asset whose resolution is still pending and assets already before the National Company Law Tribunal (NCLT). 1) Small and medium-sized enterprises (SME) resolution approach: According to report, for SME resolution a steering committee will be created by banks for formulating and validating the schemes, with a provision for additional funds. The resolution of these assets will be done by a single bank having the liberty to customize it. The resolution should be completed in 90 days. 2) Bank-led resolution approach: A bank led resolution is suggested by the committee for medium assets ranging from 50-500 crore rupees to be achieved in 180 days. The resolution plan has to be approved by lenders holding at least 66 per cent of the debt. Indian Banks Association (IBA) has to appoint a steering committee to validate the process within 30 days. Through this route, multiple banks and lenders involvement can create an issue in arriving to at a consensus. 3) AMC/AIF led resolution approach: An asset management company (AMC) will be created to deal with large loans involving 500 crores and more. Alternative investment fund (AIF) will be responsible to raise funds from institutional investors; AIF can also bid for assets in National Company Law Tribunal (NCLT). The lead bank can discover price discovery through the open auction route. Security receipts have to be redeemed within 60 days 4) NCLT/IBC approach 5) Asset-trading platform

79. Consider the following properties of ‘Neutrino’:

(1) A neutrino interacts only via weak atomic force and gravity. (2) It passes unimpeded and undetected through normal matter. (3) Neutrinos emerge from black holes and exploding stars only. Which of the properties given above is/are correct? (a) 1 and 2 only (b) 2 and 3 only (c) 1 and 3 only (d) 1, 2 and 3

Answer: (a) Explanation: Statement 1 is correct. Unlike electrons, neutrinos lack an electric charge, so the usual electromagnetic forces in space that jostle other particles do not affect them. Neutrinos roam freely in space, zipping across great distances at nearly the speed of light without slowing down or changing direction. Statement 2 is correct. They pass through planets, stars, and whole galaxies, imperceptible to all.

https://telegram.me/UPSCMaterials https://telegram.me/FreeUPSCMaterials https://telegram.me/MaterialforExam

Page 57: Answer Key & Exp for GS Prelim Test-5 held on 23rd …...1 Prelim IAS Test Series (2019) – GS Test 5 (23.12.2018) History 1 and Current Affairs (July 2018) Answer Key & Exp for GS

57

Prelim IAS Test Series (2019) – GS Test 5 (23.12.2018) History 1 and Current Affairs (July 2018)

Statement 3 is incorrect. Neutrinos emerged soon after the Big Bang, and, later on, from black holes, exploding stars, the nuclear reaction that fuels our sun, even from the interaction between cosmic radiation and Earth’s atmosphere. Source: https://www.theatlantic.com/science/archive/2018/07/neutrino-discovery-blazar-icecube/564998/ 80. With reference to Pradhan Mantri Kisan SAMPADA Yojana, which of the following

statements are correct? (1) It is being implemented under the Ministry of Food Processing Industries. (2) Mega Food Parks scheme will be implemented under SAMPADA Yojana. (3) It aims for the creation of modern infrastructure with efficient supply chain

management from farm gate to a retail outlet. Select the correct answer using the code given below: (a) 1 and 2 only (b) 2 and 3 only (c) 1 and 3 only (d) 1, 2 and 3

Answer: (d) Explanation: All the statements are correct.

The government of India (GOI) has approved a new Central Sector Scheme – Pradhan Mantri Kisan SAMPADA Yojana (Scheme for Agro-Marine Processing and Development of Agro-Processing Clusters) for the period 2016-20 coterminous with the 14th Finance Commission cycle.

The scheme will be implemented by the Ministry of Food Processing Industries (MoFPI). Pradhan Mantri Kisan SAMPADA Yojana.

PM Kisan SAMPADA Yojana is a comprehensive package which will result in the creation of modern infrastructure with efficient supply chain management from farm gate to a retail outlet.

The following schemes will be implemented under PM Kisan SAMPADA Yojana: Mega Food Parks Integrated Cold Chain and Value Addition Infrastructure Creation/ Expansion of Food Processing/ Preservation Capacities (Unit

Scheme) Infrastructure for Agro-processing Clusters Creation of Backward and Forward Linkages Food Safety and Quality Assurance Infrastructure Human Resources and Institutions Source: http://mofpi.nic.in/Schemes/pradhan-mantri-kisan-sampada-yojana 81. Which of the following are functions of the ‘Directorate General of Anti-Dumping and

Allied Duties’ (DGAD)? (1) Conducting Anti-Dumping investigations (2) Conducting Anti-subsidy (Countervailing Duty) investigations

https://telegram.me/UPSCMaterials https://telegram.me/FreeUPSCMaterials https://telegram.me/MaterialforExam

Page 58: Answer Key & Exp for GS Prelim Test-5 held on 23rd …...1 Prelim IAS Test Series (2019) – GS Test 5 (23.12.2018) History 1 and Current Affairs (July 2018) Answer Key & Exp for GS

58

Prelim IAS Test Series (2019) – GS Test 5 (23.12.2018) History 1 and Current Affairs (July 2018)

(3) Conducting Anti-Circumvention investigations (4) Defending Indian exporters from various countervailing duty investigations

carried out by foreign agencies. Select the correct answer using the code given below: (a) 1 and 2 only (b) 2 and 3 only (c) 1, 2 and 3 only (d) 1, 2, 3 and 4

Answer: (d) Explanation: Directorate General of Anti-Dumping and Allied Duties’ (DGAD) performs all the functions given above. Hence, the correct answer is option (d).

The Directorate General of Trade Remedies (earlier known as Directorate General of Anti-dumping and Allied Duties) was named in May 2018 as an integrated single window agency for providing comprehensive and swift trade defence mechanism in India.

DGTR functions as an attached office of Department of Commerce, Ministry of Commerce and Industry. It is a professionally integrated organisation with multi-spectrum skill sets emanating from officers drawn from different services and specialisations.

Earlier, the Directorate General of Anti-dumping and Allied Duties (DGAD) dealt with anti-dumping and CVD cases, Directorate General of Safeguards (DGS) dealt with safeguard measures,and DGFT dealt with quantitative restriction (QR) safeguards.

The DGTR brings DGAD, DGS and Safeguards (QR) functions of DGFT into its fold by merging them into one single national entity. DGTR now deals with Anti-dumping, CVD and Safeguard measures. It also provides trade defence support to our domestic industry and exporters in dealing with increasing instances of trade remedy investigations instituted against them by other countries.

DGTR provides a level playing field to the domestic industry against the adverse impact of the unfair trade practices like dumping and actionable subsidies from any exporting country, by using Trade Remedial methods under relevant framework of WTO arrangements, Customs Tariff Act & Rules and other relevant laws and International agreements, in a transparent and time-bound manner.

Source: http://www.dgtr.gov.in/about-us/about-department 82. Which of the following equipment(s) can be used to check the pollution generated

from coal-fired power plants? (1) Flue gas desulfurizers (2) Catalytic Reducers (3) Electrostatic Precipitators Select the correct answer using the code given below: (a) 1 only (b) 2 only (c) 2 and 3 only

https://telegram.me/UPSCMaterials https://telegram.me/FreeUPSCMaterials https://telegram.me/MaterialforExam

Page 59: Answer Key & Exp for GS Prelim Test-5 held on 23rd …...1 Prelim IAS Test Series (2019) – GS Test 5 (23.12.2018) History 1 and Current Affairs (July 2018) Answer Key & Exp for GS

59

Prelim IAS Test Series (2019) – GS Test 5 (23.12.2018) History 1 and Current Affairs (July 2018)

(d) 1, 2 and 3 Answer: (d) Explanation: Study on ‘Benefit-cost analysis of Emission Standards for coal-based thermal power plants in India’ by the Centre for Science, Technology and Policy (C-STEP), covers a greater number of plants - 263 GW of capacity till 2030 to be fitted with equipment such as flue gas desulfurisers, catalytic reducer and electrostatic precipitators. Hence, the correct answer is (d). Flue-gas desulfurization (FGD) is a set of technologies used to remove sulfur dioxide (SO2) from exhaust flue gases of fossil-fuel power plants, and from the emissions of other sulfur oxide emitting processes (e.g trash incineration) Catalytic reduction (CR) systems are post-combustion active emission control technology systems. The system uses a catalyst bed and an injected liquid-reductant agent to scavenge pollutants, nitrogen oxides (NOx), and produce diatomic nitrogen (N2) and water vapor (H2O). Electrostatic precipitator (ESP) is a filtration device that removes fine particles, like dust and smoke, from a flowing gas using the force of an induced electrostatic charge minimally impeding the flow of gases through the unit. Source: https://www.thehindubusinessline.com/news/pollution-control-equipment-in-coal-fired-power-plants-is-a-396-lakh-crore-industry/article24464066.ece 83. Which of the following factors are responsible for a high density of caves in

Meghalaya? (1) High-Grade Limestone (2) High Precipitation (3) Low solubility of sandstone (4) High Elevation (5) Humid Climate Select the correct answer using the code given below: (a) 1, 2 and 3 only (b) 2, 3 and 4 only (c) 1, 2, 4 and 5 only (d) 1, 2, 3, 4 and 5

Answer: (c) Explanation: Factors 1, 2, 4 and 5 are correct - The density of the cave systems in Meghalaya is the highest in the country, and there are a number of factors responsible for this formation: high-grade limestone, high precipitation, elevation and a humid climate. Factor 3 is incorrect - Sandstone caves are not as common as limestone caves because of the high solubility of sandstone, making it erode and dissolve much more easily than limestone. In Meghalaya, the presence of such large sandstone caves is primarily because of the prevalence of calcareous sandstone, which has a small percentage of lime, and the high rainfall. Additional Info:

https://telegram.me/UPSCMaterials https://telegram.me/FreeUPSCMaterials https://telegram.me/MaterialforExam

Page 60: Answer Key & Exp for GS Prelim Test-5 held on 23rd …...1 Prelim IAS Test Series (2019) – GS Test 5 (23.12.2018) History 1 and Current Affairs (July 2018) Answer Key & Exp for GS

60

Prelim IAS Test Series (2019) – GS Test 5 (23.12.2018) History 1 and Current Affairs (July 2018)

The world's longest sandstone cave named Krem Puri was recently discovered (2018) in Meghalaya. It has a staggering length of 24.5 km, almost three times the height of Mount Everest, and contains some dinosaur fossils from 66-76 million years ago as well. 'Krem' means 'cave' in Khasi language. The cave system which is 24,583 metres long was discovered in near Laitsohum village, located in the Mawsynram area in Meghalaya's East Khasi Hills district. Though Krem Puri had been discovered in 2016 itself, its actual length was mapped only recently when the Meghalaya Adventurers' Association (MAA) took a 25-day expedition to this end. Source: https://timesofindia.indiatimes.com/india/human-greed-may-wipe-out-meghalayas-caves/articleshow/65047263.cms 84. Consider the following statements:

(1) The International Commission on Stratigraphy is responsible for standardising the geological timescale.

(2) The International Commission on Stratigraphy works under International Union of Geological Sciences (IUGS).

(3) Humans are currently living in the Meghalaya subdivision of the Holocene epoch.

Which of the statements given above are correct? (a) 1 and 2 only (b) 2 and 3 only (c) 1 and 3 only (d) 1, 2 and 3

Answer: (d) Explanation: Stratigraphy is a branch of geology concerned with the study of rock layers (strata) and layering (stratification). It is primarily used in the study of sedimentary and layered volcanic rocks. Stratigraphy has two related subfields: lithostratigraphy (lithologic stratigraphy) and biostratigraphy (biologic stratigraphy). Statement 1 is correct - The International Commission on Stratigraphy, or ICS, is the global governing body that formally names geological eras, associating each rock layer with a specific period. For instance, the ICS formally named and identified the Jurassic, to the immense benefit of palaeontologists and fictional theme-park developers everywhere. Statement 2 is correct. The International Union of Geological Sciences, the global governing body for geologists, reviews the proposals submitted by International Commission on Stratigraphy regarding geological timescales and approve it. Statement 3 is correct. International Commission on Stratigraphy announced that the current stretch of geological time, the Holocene Epoch, would be split into three subdivisions. Thus, though we live in Holocene epoch, we also live in the youngest of these new subdivisions: the Meghalayan Age. Source: https://www.theatlantic.com/science/archive/2018/07/anthropocene-holocene-geology-drama/565628/

https://telegram.me/UPSCMaterials https://telegram.me/FreeUPSCMaterials https://telegram.me/MaterialforExam

Page 61: Answer Key & Exp for GS Prelim Test-5 held on 23rd …...1 Prelim IAS Test Series (2019) – GS Test 5 (23.12.2018) History 1 and Current Affairs (July 2018) Answer Key & Exp for GS

61

Prelim IAS Test Series (2019) – GS Test 5 (23.12.2018) History 1 and Current Affairs (July 2018)

85. The Parker Project is in news sometimes. It is related to which of the following? (a) A project to study the outer corona of the Sun. (b) A manned mission to Mars (c) A project by the NASA on artificial intelligence. (d) A project designed to bring Internet connectivity to rural and remote

communities worldwide. Answer: (a) Explanation: Parker Solar Probe is a NASA robotic spacecraft En route to probe the outer corona of the Sun. It will approach to within 9.86 solar radii (6.9 million kilometres) from the centre of the Sun and will travel, at closest approach, as fast as 690,000 km/h. The project was announced in 2009. The Johns Hopkins University Applied Physics Laboratory designed and built the spacecraft, which was launched on August 12, 2018. It became the first NASA spacecraft named after a living person, honouring physicist Eugene Parker, professor emeritus at the University of Chicago. The solar wind is a stream of charged particles released from the upper atmosphere of the Sun called the corona. Additional Information; How it affects Earth? It affects it by the intense clouds of high energy particles that it often contains which are produced by solar storms. When these clouds, called coronal mass ejections, make their way to the Earth in 3-4 days, they collide with the magnetic field of the Earth and cause it to change its shape. The particles then leak through the magnetic field of the Earth, particularly near the north and south poles, and cause still more changes to the magnetic field of the Earth, this time at even lower altitudes closer to the ground. These changes can produce many problems with electrical equipment. The way in which solar wind 'plasma' invades the Earth's magnetic field and seeps into the inner regions where the Van Allen radiation belts are located, is not very well known. Also, in the direction opposite the Sun, the Earth's magnetic field is pulled way out into interplanetary space making it look like a comet. In this 'geotail' region many different electrical disturbances take place that can accelerate particles to very high speeds and energies. All of this is made much more violent by the solar wind, especially the storm clouds that the Sun launches our way from time to time! Source: https://image.gsfc.nasa.gov 86. Consider the following statements in the context of the ‘Adopt a Heritage’ scheme:

(1) It is an initiative of the Ministry of Tourism. (2) Under this scheme only public sector companies are invited to develop

selected monuments and heritage and tourist sites. Which of the statement(s) given above is/are correct? (a) 1 only (b) 2 only (c) Both 1 and 2 (d) Neither 1 nor 2

https://telegram.me/UPSCMaterials https://telegram.me/FreeUPSCMaterials https://telegram.me/MaterialforExam

Page 62: Answer Key & Exp for GS Prelim Test-5 held on 23rd …...1 Prelim IAS Test Series (2019) – GS Test 5 (23.12.2018) History 1 and Current Affairs (July 2018) Answer Key & Exp for GS

62

Prelim IAS Test Series (2019) – GS Test 5 (23.12.2018) History 1 and Current Affairs (July 2018)

Answer: (a) Explanation: Statement (1) is correct. “Adopt a Heritage: Apni Dharohar, Apni Pehchaan”, is a collaborative effort between the Ministry of Tourism, Ministry of Culture and Archaeological Survey of India (ASI), and State/UTs Governments. The Project aims to develop synergy among all partners to effectively promote “responsible tourism”. Statement (2) is incorrect. It aims to involve public sector companies, private sector companies and corporate citizens/individuals to take up the responsibility for making our heritage and tourism more sustainable through development, operation and maintenance of world-class tourist infrastructure and amenities at ASI/ State heritage sites and other important tourist sites in India. They would become ‘Monument Mitras’ through the innovative concept of “Vision Bidding”, where the agency with best vision for the heritage site will be given an opportunity to associate pride with their CSR activities. They would also get limited visibility in the premises and the Incredible India website. The aim is enhancement of tourist experience and promotion of the incredible heritage sites to bring them on tourism map. 87. The term CAATSA was in news recently. Which of the following option is correct

description of the CAATSA? (a) It is UN convention for protection of rights of refugees. (b) It is EU’s data protection law for its citizen. (c) It is a UN’s law to curb the menace of global terrorism (d) It is US Federal law to counter American Adversaries through sanctions

Answer: (d) Explanation: The Countering America's Adversaries Through Sanctions Act (CAATSA), is a United States federal law that imposed sanctions on Iran, North Korea, and Russia. Likely Impact on India: There are only two sanctions that may impact either India-Russia relations or India-US relations. The first of these, which is likely to have an impact on India-Russia relations, is the “Prohibition of Banking transactions." Under this, the Secretary of the Treasury would prohibit the opening, and prohibit or impose strict conditions on maintaining, in the United States of correspondent accounts or payable-through accounts of Financial Institutions engaged in transfer of funds from the sanctioned person to the Russian defence sector. This would mean difficulties for India in making payments in US Dollars to Rosoboronexport for the purchase of the S-400 systems. The second sanction will have far greater consequences, not for India-Russia relations, but for India-US relations. And that is the “Export sanction, which has the potential to completely derail the India-US Strategic and Defence partnership, as it will deny the license for, and export of, any items controlled by the US to the sanctioned person.

https://telegram.me/UPSCMaterials https://telegram.me/FreeUPSCMaterials https://telegram.me/MaterialforExam

Page 63: Answer Key & Exp for GS Prelim Test-5 held on 23rd …...1 Prelim IAS Test Series (2019) – GS Test 5 (23.12.2018) History 1 and Current Affairs (July 2018) Answer Key & Exp for GS

63

Prelim IAS Test Series (2019) – GS Test 5 (23.12.2018) History 1 and Current Affairs (July 2018)

This will not, however, bar India from obtaining spares and replacements for the items already in the Indian inventory. 88. Consider the following pairs:

Places in News Location

1. Sakhalin Island : Russia

2. Baltic Sea : Atlantic Ocean

3. Pyrenees Mountains : Europe

Which of the pairs given above is/are correctly matched? (a) 1 only (b) 2 only (c) 2 and 3 only (d) 1, 2 and 3

Answer: (d) Explanation: Sakhalin is a large Russian island in the North Pacific Ocean. It is Russia's largest island located just off the east coast of Russia, and just north of Japan. Russia has built new troop barracks and plans more military infrastructure on a disputed Pacific island chain, angering Japan before potential high-level negotiations. Russia is also building heated garages for more than 100 armoured vehicles there and will put up three more military dormitories next year. The Japanese foreign ministry said it would file a note of protest over the construction on two of the southern islands, which were occupied by the Soviet Union at the end of the Second World War but are still claimed by Tokyo. The Baltic Sea is a marginal sea of the Atlantic Ocean, enclosed by Finland, Sweden, Denmark, Estonia, Latvia, Lithuania, northwest Russia, Poland, Germany and the North and Central European Plain. Innovation stakeholders in the Baltic Sea region, gathered at a conference organised by the Baltic Science Network and the Baltic TRAM project in Brussels, made their case for strengthened cooperation between research and industry which could help better exploit the economic value of big science labs and mitigate the effects of brain drain. The Pyrenees is a range of mountains in southwest Europe that forms a natural border between Spain and France. Reaching a height of 3,404 meters altitude at the peak of Aneto, the range separates the Iberian Peninsula from the rest of continental Europe. 89. Consider the following statements regarding Mars Express Mission:

(1) Mars Express Mission is launched by the European Space Agency. (2) The mission has detected liquid water buried under layers of ice and dust in

the south polar region of Mars. Which of the statements given above is/are correct? (a) 1 only (b) 2 only (c) Both 1 and 2

https://telegram.me/UPSCMaterials https://telegram.me/FreeUPSCMaterials https://telegram.me/MaterialforExam

Page 64: Answer Key & Exp for GS Prelim Test-5 held on 23rd …...1 Prelim IAS Test Series (2019) – GS Test 5 (23.12.2018) History 1 and Current Affairs (July 2018) Answer Key & Exp for GS

64

Prelim IAS Test Series (2019) – GS Test 5 (23.12.2018) History 1 and Current Affairs (July 2018)

(d) Neither 1 nor 2 Answer: (c) Explanation: Statements (1) and (2) both are correct. Mars Express is a space exploration mission being conducted by the European Space Agency (ESA). The Mars Express mission is exploring the planet Mars, and is the first planetary mission attempted by the agency. Mars Express consists of two parts, the Mars Express Orbiter and Beagle 2, a lander designed to perform exobiology and geochemistry research. The mission has detected liquid water buried under layers of ice and dust in the south polar region of Mars. Recently, the Mars Express has been granted mission extension, until the end of 2020. 90. Which of the following is/are not the recommendations of B N Srikrishna Committee

on data privacy? (1) Amending Aadhaar Act to tighten data protection and to empower UIDAI to act

against companies misusing information. (2) All firms have to appoint a Data Protection Officer. (3) Data protection authority of India shall be set by GOI. Select the correct answer using the code given below: (a) 2 only (b) 2 and 3 only (c) 1 and 3 only (d) None of the above

Answer: (d) Explanation: All are the recommendations given by B N Srikrishna Committee on data privacy Recognizing the importance of data protection and keeping personal data of citizens secure and protected, Ministry of Electronics and Information Technology (MeitY), Government of India had, on 31st July 2017, constituted a Committee of Experts under the Chairmanship of Justice B N Srikrishna, Former Judge, Supreme Court of India to study and identify key data protection issues and recommend methods for addressing them. Key Recommendations: Data Protection Authority: The committee calls for the creation of an independent data protection authority, to oversee and, if needed, enforce India's data protection regime. The DPA shall be appointed by a selection committee comprising of the Chief Justice of India, government nominees, and an independent expert. Consent: The committee recognizes individual consent as "the bulwark on which data processing practices in the digital economy are founded", even as it notes that the consent framework is hopelessly broken as denial of consent entails a denial of service. Hence the committee proposes that consent contracts — such as the terms and conditions that internet users routinely sign without reading — must themselves only

https://telegram.me/UPSCMaterials https://telegram.me/FreeUPSCMaterials https://telegram.me/MaterialforExam

Page 65: Answer Key & Exp for GS Prelim Test-5 held on 23rd …...1 Prelim IAS Test Series (2019) – GS Test 5 (23.12.2018) History 1 and Current Affairs (July 2018) Answer Key & Exp for GS

65

Prelim IAS Test Series (2019) – GS Test 5 (23.12.2018) History 1 and Current Affairs (July 2018)

collect data that is relevant to the services provided, and must not be shared in a manner not reasonably expected by an individual. Limited Storage: The committee recommends that data should be stored for the time required to fulfill the purpose for which it was collected, and should subsequently be erased. The Right to Be Forgotten: The report considers the possibility of allowing for the right for a person to ask for personal data, collected by any entity, to be erased after a process of adjudication by the Data Protection Authority. Cross-Border Movement of Data: The movement of data across national borders is likely to have the biggest impact on technology companies working in India. The report proposes a segregated approach that categorizes data on the basis of its sensitivity — while highly sensitive data shall have to stay within India, in most cases, the report allows for companies to maintain at least one serving copy of the data in India. 91. Consider the following statements with respect to ‘Smartnet’:

(1) Smartnet is an initiative of the Ministry of Electronics and Information Technology to support the development of cities across India.

(2) Smartnet provides a horizontal learning and knowledge sharing platform for exchange between cities, practitioners, academia, researchers and technologists.

Which of the statement(s) given above is/are correct? (a) 1 only (b) 2 only (c) Both 1 and 2 (d) Neither 1 nor 2

Answer: (b) Explanation: Smartnet is an initiative of the Ministry of Housing and Urban Affairs to support the development of cities across India and to create a resource-rich ecosystem of learning, sharing and disseminating for city managers and primary stakeholders in the urban transformation of India. It will provide a horizontal learning and knowledge sharing platform for exchange between cities, practitioners, academia, researchers and technologists. 92. Consider the following statements with respect to Data Security Council of India

(DSCI) (1) It is a not-for-profit, industry body on data protection in India. (2) The objective of DSCI is to make the cyberspace safe, secure and trusted by

establishing best practices in cyber security and privacy. Which of the statement(s) given above is/are correct? (a) 1 only (b) 2 only (c) Both 1 and 2 (d) Neither 1 nor 2

https://telegram.me/UPSCMaterials https://telegram.me/FreeUPSCMaterials https://telegram.me/MaterialforExam

Page 66: Answer Key & Exp for GS Prelim Test-5 held on 23rd …...1 Prelim IAS Test Series (2019) – GS Test 5 (23.12.2018) History 1 and Current Affairs (July 2018) Answer Key & Exp for GS

66

Prelim IAS Test Series (2019) – GS Test 5 (23.12.2018) History 1 and Current Affairs (July 2018)

Answer: (c) Explanation: Both statements are correct. Data Security Council of India (DSCI), is a not-for-profit, industry body on data protection in India, setup by NASSCOM, committed to making the cyberspace safe, secure and trusted by establishing best practices, standards and initiatives in cyber security and privacy. To further its objectives, DSCI engages with governments and their agencies, regulators, industry sectors, industry associations and think tanks for policy advocacy, thought leadership, capacity building and outreach activities. 93. UIDAI introduced a new concept of 'Virtual ID'. With reference to this concept, which

of the following statements is/are correct? (1) Aadhaar holder can regenerate Virtual ID multiple times in a day. (2) Only Virtual ID is sufficient to authenticate transactions and e-know-your-

customer (KYC) services of Aadhaar holder. (3) One's Aadhaar number can be derived from the Virtual ID generated. Select the correct answer using the code given below: (a) 1 only (b) 1 and 2 only (c) 2 and 3 only (d) 1, 2 and 3

Answer: (a) Explanation: UIDAI introduced a new concept of 'Virtual ID' which Aadhaar-card holder can generate from its website. Aadhaar number holder can use Virtual ID in lieu of Aadhaar number. The move aims to strengthen the privacy and security of Aadhaar data and comes amid heightened concerns around the collection and storage of personal and demographic data of individuals. Statement 1 is correct. Since this is a digital ID, Aadhaar holders can regenerate it multiple times, which makes it safer than providing your actual Aadhaar number. At present, the VID is valid for a minimum of one day, which means an Aadhaar holder would be able to re-generate a new VID a day after he has generated the first one. Further, there is no expiry period defined yet for the VID and it will be valid till the time a new one is generated by the Aadhaar card holder. A user can generate as many Virtual IDs as he or she wants. The older ID gets automatically cancelled once a fresh one is generated Statement 2 is incorrect. One's Aadhaar number cannot be derived from the VID that is generated. Statement 3 is incorrect. The Virtual ID can be used for the purpose of authentication in the same way the Aadhaar number is used.The Virtual ID, which would be a random 16-digit number, together with biometrics of the user would give any authorised agency like a mobile company, limited details like name, address and photograph, which are enough for any verification.

https://telegram.me/UPSCMaterials https://telegram.me/FreeUPSCMaterials https://telegram.me/MaterialforExam

Page 67: Answer Key & Exp for GS Prelim Test-5 held on 23rd …...1 Prelim IAS Test Series (2019) – GS Test 5 (23.12.2018) History 1 and Current Affairs (July 2018) Answer Key & Exp for GS

67

Prelim IAS Test Series (2019) – GS Test 5 (23.12.2018) History 1 and Current Affairs (July 2018)

Source: https://economictimes.indiatimes.com/wealth/personal-finance-news/what-is-aadhaar-virtual-id-and-how-you-can-generate-one/articleshow/63527063.cms 94. Recently, India’s nomination of the "Victorian and Art Deco Ensembles of Mumbai"

has been inscribed on UNESCO's World Heritage list. In the past 5 years alone, India has managed to get inscribed seven of its properties/sites on the World Heritage List of UNESCO. India now has overall 37 World Heritage Inscriptions with 29 Cultural, 07 Natural and 01 Mixed sites. Which one of the following countries has maximum number of UNESCO’s World Heritage properties in ASPAC (Asia and Pacific) region? (a) India (b) Italy (c) China (d) Mexico

Answer: (c) Explanation: While India stands second largest in number after China in terms of number of World Heritage properties in ASPAC (Asia and Pacific) region, it is overall sixth in the world. This achievement is expected to give a tremendous fillip to domestic and international tourism leading to increased employment generation, creation of world-class infrastructure and augmentation of sale of local handicrafts, handlooms and heritage memorabilia. Italy takes the crown as the country with the most UNESCO World Heritage Sites in the world. A late starter to the UNESCO game, China has powered up the UNESCO ranks with 52 sites now listed. When excluding UNESCOs shared with other countries, China is the country with the highest number of unique sites. A mix of nature and culture mean Mexico has been awarded 34 UNESCO World Heritage Site listings, the highest number in South America and the 7th in the world. Source: http://pib.nic.in/newsite/PrintRelease.aspx?relid=180317 95. Which of the following crops are considered as 'Nutri-Cereals’?

(1) Rice (2) Bajra (3) Wheat (4) Ragi Select the correct answer using the code given below: (a) 1 and 2 only (b) 2 and 3 only (c) 2 and 4 only (d) 2, 3 and 4 only

Answer: (c) Explanation: The Central Government has renamed jowar, bajra, ragi and other millets as “Nutri Cereals”, dispensing with the nomenclature “coarse cereals”. The move is aimed at removing a lingering

https://telegram.me/UPSCMaterials https://telegram.me/FreeUPSCMaterials https://telegram.me/MaterialforExam

Page 68: Answer Key & Exp for GS Prelim Test-5 held on 23rd …...1 Prelim IAS Test Series (2019) – GS Test 5 (23.12.2018) History 1 and Current Affairs (July 2018) Answer Key & Exp for GS

68

Prelim IAS Test Series (2019) – GS Test 5 (23.12.2018) History 1 and Current Affairs (July 2018)

perception that these grains are inferior to rice and wheat, even as their health benefits are larger. The Ministry of Agriculture notified millets comprising Sorghum (Jowar), Pearl Millet (Bajra), Finger Millet (Ragi/Mandua), Minor Millets - Foxtail Millet (Kangani/Kakun), Proso Millet (Cheena), Kodo Millet (Kodo), Barnyard Millet (Sawa/Sanwa/ Jhangora), Little Millet (Kutki) and two Pseudo Millets (Black-wheat (Kuttu) and Ameranthus (Chaulai) which have high nutritive value as “Nutri Cereals” for production, consumption and trade point of view. The Food and Agriculture Organisation (FAO) Council in Rome approved India’s proposal to observe an International Year of Millets in 2023. This will enhance global awareness to bring back these nutri-cereals to the plate, for food and nutrition security and hence increase production for resilience to challenges posed globally by climate change. This is further supported by increase in Minimum Support Prices (MSP) of millets. Millets consists of Jowar, Bajra, Ragi and minor millets together termed as nutri-cereals. 96. Recently, NITI Aayog has recommended the use of HCNG as an alternative fuel.

Which of the following statements is/are correct about HCNG fuel? (1) It is a blend of hydrogen and CNG, the ideal hydrogen concentration being

50%. (2) It is more efficient and environment friendly than CNG. (3) It produces only water and heat instead of emissions making it the cleanest

fuel. Select the correct answer using the code given below: (a) 2 only (b) 1 and 3 only (c) 2 and 3 only (d) 1, 2 and 3

Answer: (a) Explanation: Statement 1 is incorrect. H-CNG is a blend of hydrogen and CNG, the ideal hydrogen concentration being 18%. Statement 2 is correct. It is more efficient and environment friendly than CNG. Statement 3 is incorrect. Compared to conventional CNG, use of H-CNG can reduce emission of carbon monoxide up to 70%. The engines of CNG-fuelled vehicles will be able to process hythane or H-CNG considering the ratio at which hydrogen is being mixed.While recommending the use of H-CNG as an alternative fuel, the NITI Aayog-CII Action Plan for Clean Fuel notes that physical blending of CNG and hydrogen involves a series of energy-intensive steps that would make H-CNG more expensive than CNG. Source: https://timesofindia.indiatimes.com/city/delhi/better-than-cng-your-bus-may-be-more-fuel-efficient-emit-less-carbon/articleshow/64890576.cms 97. Camp Lemonnier was in news recently. It is _________

(a) World’s biggest refugee camp located in Lebanon (b) An organization creating awareness about # Me Too campaign (c) New missile system procured by India from Russia

https://telegram.me/UPSCMaterials https://telegram.me/FreeUPSCMaterials https://telegram.me/MaterialforExam

Page 69: Answer Key & Exp for GS Prelim Test-5 held on 23rd …...1 Prelim IAS Test Series (2019) – GS Test 5 (23.12.2018) History 1 and Current Affairs (July 2018) Answer Key & Exp for GS

69

Prelim IAS Test Series (2019) – GS Test 5 (23.12.2018) History 1 and Current Affairs (July 2018)

(d) Naval Base located near Gulf of Aden Answer: (d) Explanation: Camp Lemonnier is the US Naval base in Djibouti. Djibouti, an arid Horn of Africa nation,

also has become a military outpost for China, France, Italy and Japan. Source: https://www.bbc.com/news/world-africa-33115502 98. The ‘Iron Dome’, an anti-missile system belongs to

(a) Iran (b) Russia (c) USA (d) Israel

Answer: (d) Explanation: The Iron Dome missile defense system, designed and developed by Israel and jointly funded through the United States, is a response to the threats Israel faces from short and medium-range rockets and mortar shells fired by Palestinian terrorists in Gaza. The system has the capability to identify and destroy such projectiles before they land in Israeli territory and is considered one of the most effective anti-missile systems in the world. Source: https://www.timesofisrael.com/rocket-alert-sirens-blare-in-southern-israel-as-idf-strikes-gaza/ 99. Consider the following statements:

(1) Guar gum is derived from guar seeds which grow in the semi-arid region of India.

(2) One of its growing applications is in hydraulic fracking of shale gas (3) India is a leading exporter of guar gum. Which of the statements given above is/are correct? (a) 1 only (b) 2 and 3 only (c) 1 and 3 only (d) 1, 2 and 3

Answer: (d) Explanation: Statement 1 is correct. Guar gum is derived from guar seeds (cluster beans), a legume crop that grows in semi-arid regions of Rajasthan, Gujarat and Haryana. Statement 2 is correct. Drilling companies use it to thicken water that is mixed with sand and pumped through shale rock cracks to extract gas. It is also used as thickening agent in products such as ice-cream, bread, pasta, sausages and pastries.

https://telegram.me/UPSCMaterials https://telegram.me/FreeUPSCMaterials https://telegram.me/MaterialforExam

Page 70: Answer Key & Exp for GS Prelim Test-5 held on 23rd …...1 Prelim IAS Test Series (2019) – GS Test 5 (23.12.2018) History 1 and Current Affairs (July 2018) Answer Key & Exp for GS

70

Prelim IAS Test Series (2019) – GS Test 5 (23.12.2018) History 1 and Current Affairs (July 2018)

Statement 3 is correct. India is a leading exporter of guar gum, commanding about 80% of global production, followed by Pakistan. Source: https://economictimes.indiatimes.com/markets/commodities/news/indian-guar-gum-on-a-high-as-us-steps-up-drilling/articleshow/57201625.cms 100. Which of the following statements is/are correct regarding ‘Zero Budget Natural

Farming’? (1) The inputs used for seed treatment are locally available in the form of cow

dung and cow urine. (2) It protects soil from degradation, helps in retaining soil fertility and is climate

change resilient. Select the correct answer using the code given below: (a) 1 only (b) 2 only (c) Both 1 and 2 (d) Neither 1 nor 2

Answer: (c) Explanation: Statement 1 and 2 are correct. Zero Budget Natural Farming (ZBNF) is a set of farming methods, and also a grassroots peasant movement, which has spread to various states in India. It has attained wide success in southern India, especially the southern Indian state of Karnataka where it first evolved. Privatized seeds, inputs, and markets are inaccessible and expensive for peasants. Indian farmers increasingly find themselves in a vicious cycle of debt, because of the high production costs, high interest rates for credit, the volatile market prices of crops, the rising costs of fossil fuel based inputs, and private seeds. Debt is a problem for farmers of all sizes in India. Under such conditions, ‘zero budget’ farming promises to end a reliance on loans and drastically cut production costs, ending the debt cycle for desperate farmers. The word ‘budget’ refers to credit and expenses, thus the phrase 'Zero Budget' means without using any credit, and without spending any money on purchased inputs. 'Natural farming' means farming with Nature and without chemicals. Additional Info: The Government of Andhra Pradesh has launched a scale-out plan for transition of 6 million farms/farmers cultivating 8 million hectares of land from conventional synthetic chemical agriculture to Zero-Budget Natural Farming (ZBNF) by 2024, making Andhra Pradesh India’s first 100 per cent natural farming state. The programme is a contribution towards the UN Sustainable Development Goals, focusing on ‘No Poverty’, ‘Clean Water and Sanitation’, ‘Responsible Consumption and Production’, and ‘Life on Land’. It is led by Rythu Sadhikara Samstha (RySS) – a not-for-profit established by the Government to implement the ZBNF programme - and supported by the Sustainable India Finance Facility (SIFF) – an innovative partnership between UN Environment, BNP Paribas, and the World Agroforestry Centre.

https://telegram.me/UPSCMaterials https://telegram.me/FreeUPSCMaterials https://telegram.me/MaterialforExam